You are on page 1of 119

Q001.- Most common pituitary tumor? % of all pituitary tumors?

A001.- Prolactinoma with hyperprolactinemia makes up 30% of pituitary tumors.

Not for Resale & redistribution. Copyright joenny_king at www.ebay.com.


Q002.- Characteristics of prolactinoma?

A002.- Most common pituitary tumor- Chromophobic staining- In women, leads to am


enorrhea, galactorrhea- Undergoes dystrophic calcification- Underlying cause for
1/4 cases of amenorrhea

Not for Resale & redistribution. Copyright joenny_king at www.ebay.com.


Q003.- Treatment for prolactinoma?

A003.- Most commonly bromocriptine: a dopamine receptor agonist that causes the
lesion to shrink- Surgery

Not for Resale & redistribution. Copyright joenny_king at www.ebay.com.


Q004.- What hypothalamic factors control release of the following pituitary horm
ones: TSH, PRL, ACTH, GH, FSH, LH?- Is the controlling factor stimulatory or inh
ibitory for each?

A004.- TSH: stimulated by TRH, inhibited by somatostatin- Prolactin: inhibited b


y dopamine- ACTH: stimulated by CRH- GH: stimulated by GHRH, inhibited by somato
statin- FSH: stimulated by GnRH- LH: stimulated by GnRH

Not for Resale & redistribution. Copyright joenny_king at www.ebay.com.


Q005.- Characteristics of somatotropic adenoma?

A005.- Second most common pituitary tumor- Acidophilic staining- Causes release
of somatomedins by liver (IGF-I, somatomedin C)- Causes gigantism if during chil
dhood, acromegaly during adulthood.- Causes local compression in sela turcica.

Not for Resale & redistribution. Copyright joenny_king at www.ebay.com.


Q006.- Characteristics of corticotropic adenoma?

A006.- Hypercorticism (Cushing disease)- Basophilic staining- Weight gain, moon


facies, thin/atrophied skin, hirsuitism, HTN, hyperglycemia due to insulin resis
tance

Not for Resale & redistribution. Copyright joenny_king at www.ebay.com.


Q007.- Difference between Cushing disease and Cushing syndrome?

A007.- "Syndrome" is hypercorticism regardless of cause, "disease" is in the cas


e of corticotropic adenoma of the pituitary.

Not for Resale & redistribution. Copyright joenny_king at www.ebay.com.


Q008.- What is pituitary cachexia (Simmonds disease)? What are the 2 most common
causes?

A008.- Generalized panhypopituitarism leading to marked wasting.- Pituitary tumo


rs and Sheehan's syndrome (postpartum pituitary necrosis) are the most common ca
uses.

Not for Resale & redistribution. Copyright joenny_king at www.ebay.com.


Q009.- What is Sheehan's syndrome?

A009.- Ischemic necrosis of the pituitary during child birth, due to hypoperfusi
on.- The pituitary enlarges during pregancy but its blood supply does not... hen

ce it is suspectible to peripartum blood loss.

Not for Resale & redistribution. Copyright joenny_king at www.ebay.com.


Q010.- Symptoms of growth hormone deficiency?

A010.- In children: growth retardation- In adults: increased insulin sensitivity


with hypoglycemia, decreased muscle strength, and anemia.

Not for Resale & redistribution. Copyright joenny_king at www.ebay.com.


Q011.- Symptoms of gonadotropin deficiency?

A011.- In children: retarded sexual maturation- In adult men: loss of libido/mus


cular mass, impotence, decreased body hair- In adult women: amenorrhea, vaginal
atrophy

Not for Resale & redistribution. Copyright joenny_king at www.ebay.com.


Q012.- Result of TSH deficiency?

A012.- Secondary hypothyroidism.

Not for Resale & redistribution. Copyright joenny_king at www.ebay.com.


Q013.- How to distinguish between deficiency of ACTH and primary adrenal failure
(Addison disease)?

A013.- In Addison disease B-melanocyte stimulating hormone is still high leading


to hyperpigmentation of skin. - Not true in ACTH deficiency, where B-MSH is als
o low.

Not for Resale & redistribution. Copyright joenny_king at www.ebay.com.

Q014.- Function of oxytocin?

A014.- Induces uterine contraction during labor, ejection of milk from mammary a
lveoli

Not for Resale & redistribution. Copyright joenny_king at www.ebay.com.


Q015.- Most common cause of SIADH?

A015.- Small cell carcinoma of lung.- Other tumors can also produce ectopic ADH.

Not for Resale & redistribution. Copyright joenny_king at www.ebay.com.


Q016.- Causes of ADH deficiency (diabetes insipidus)?

A016.- Tumors- Trauma- Inflammatory processes- Lipid storage disorders- etc.

Not for Resale & redistribution. Copyright joenny_king at www.ebay.com.


Q017.- What is empty sella syndrome?

A017.- Pituitary hypofunction due to destruction of all or part of the pituitary


.

Not for Resale & redistribution. Copyright joenny_king at www.ebay.com.


Q018.- What is Nelson syndrome?

A018.- Large pituitary adenomas that develop after bilateral adrenalectomy: due
to a loss of feedback inhibition on preexisting pituitary microadenomas.

Not for Resale & redistribution. Copyright joenny_king at www.ebay.com.


Q019.- What hormones are released by the neurohypophysis? Is it anterior or post
erior?

A019.- Oxytocin, ADH- Posterior

Not for Resale & redistribution. Copyright joenny_king at www.ebay.com.


Q020.- What hormones are released by the adenohypophysis? Is it anterior or post
erior?

A020.- TSH, prolactin, ACTH, GH, FSH, LH- Anterior

Not for Resale & redistribution. Copyright joenny_king at www.ebay.com.


Q021.- What is a craniopharyngioma?

A021.- Benign childhood tumor from remnants of Rathke pouch.- Often cystic or ca
lcified.- Not a true pituitary tumor, but can have mass effects that cause pitui
tary hypofunction.

Not for Resale & redistribution. Copyright joenny_king at www.ebay.com.


Q022.- What are the symptoms of a nonsecreting pituitary adenoma or other mass l
esion in the sella?

A022.- Hypopituitarism, - headache, - visual disturbances (bilateral hemianopsia


due to optic chiasm pressure) - and palsies (due to cranial nerve compression)

Not for Resale & redistribution. Copyright joenny_king at www.ebay.com.


Q023.- What is the most common congenital anomaly of the thyroid? other anomalie
s?

A023.- thyroglossal duct cyst; it does not lead to alterations in thyroid functi
on.- Ectopic thyroid tissue may also be found anywhere along the course of the t
hyroglossal duct. - (If removing these, be sure they are not the ONLY thyroid ti
ssue!)

Not for Resale & redistribution. Copyright joenny_king at www.ebay.com.


Q024.- what are the causes of goiter?

A024.- physiologic enlargement during puberty and pregnancy- iodine deficiencyHashimoto thyroiditis- goitrogens: substances that suppress synthesis of thyroid
hormones- dyshormonogenesis: failure of hormone synthesis due to enzyme defects

Not for Resale & redistribution. Copyright joenny_king at www.ebay.com.


Q025.- what is the difference between a hot nodule and cold nodule in the thyroi
d?

A025.- most nodules are hypoplastic and do not take up radioactive iodine (cold)
- Some nodules are hyperplastic and actively produce thyroid hormone, and take u
p radioactive iodine (hot)- Nonfunctional (cold) nodules are more likely to be m
alignant compared to functional (hot) nodules.

Not for Resale & redistribution. Copyright joenny_king at www.ebay.com.


Q026.- what are the causes and characteristics of myxedema?

A026.- Therapy for hyperthyroidism, Hashimoto thyroiditis, idiopathic, iodine de


ficiency- Clinical characteristics include cold intolerance, weight gain, low vo
ice, mental slowness, menorrhagia, constipation, dry skin, hair loss, puffiness
of the face

Not for Resale & redistribution. Copyright joenny_king at www.ebay.com.


Q027.- what are the causes and characteristics of cretinism?

A027.- Iodine deficiency, thyroid enzyme deficiency, poor thyroid development, f


ailure of fetal thyroid dissent, trans placental transfer of antithyroid antibod
ies from a mother with autoimmune thyroid disease- Characteristics include sever
e mental retardation, impaired physical growth and dwarfism, a large tongue, and
a protuberant abdomen

Not for Resale & redistribution. Copyright joenny_king at www.ebay.com.


Q028.- what are the symptoms of hyperthyroidism (thyrotoxicosis)?

A028.- Restlessness, - irritability, - fatigability, - tremor, heat intolerance


and sweating, - tachycardia, - muscle wasting and weight loss, - fine hair, diar
rhea, - menstrual abnormalities, - and most importantly greatly increased T4.

Not for Resale & redistribution. Copyright joenny_king at www.ebay.com.


Q029.- what is Graves' disease? What is its mechanism? Susceptible individuals?

A029.- Hyperthyroidism caused by a diffuse toxic goiter. - It occurs more freque


ntly in women than in men, and the incidence is increased in HLA-DR3 and HLA-B8
positive individuals. - Exophthalmos is characteristic and unique.- Autoimmune d
isease, with thyroid stimulating immunoglobulin (TSI) and thyroid growth immunog
lobulin (TGI) causing hormone production and hyperplasia.

Not for Resale & redistribution. Copyright joenny_king at www.ebay.com.


Q030.- what are the three types of thyroiditis?

A030.- Hashimoto thyroiditis- subacute granulomatous thyroiditis- Riedel thyroid


itis

Not for Resale & redistribution. Copyright joenny_king at www.ebay.com.


Q031.- What is Hashimoto thyroiditis?- Who is susceptible?

A031.- Autoimmune disorder, common cause of hypothyroidism. - Histologically, ma


ssive infiltrates of lymphocytes with germinal center formation and Hurtle cells
. - Autoantibodies are present, including anti-thyroglobulin, antithyroid peroxi
dase, anti-TSH receptor, and anti-iodine receptor.- Associated with increased in
cidence of pernicious anemia, diabetes mellitus, and Sjogren's syndrome. - Assoc
iated with HLA-DR5 and HLA-B5.

Not for Resale & redistribution. Copyright joenny_king at www.ebay.com.


Q032.- What is subacute granulomatous thyroiditis? What appears to be a common c
ause?

A032.- focal destruction of thyroid tissue and granulomatous inflammation caused


by a variety of viral infections, including mumps and Coxsackie virus.- Follows
a self-limiting course of several weeks duration consisting of a flulike illnes
s, pain and tenderness of the thyroid, and a transient hyperthyroidism.

Not for Resale & redistribution. Copyright joenny_king at www.ebay.com.


Q033.- what is Riedel thyroiditis?

A033.- Idiopathic replacement of the thyroid by fibrous tissue, can mimic carcin
oma.

Not for Resale & redistribution. Copyright joenny_king at www.ebay.com.


Q034.- What are the characteristics of papillary carcinoma of the thyroid? Commo
n causes?

A034.- the most common thyroid cancer- Histologically: papillary projection into
gland-like spaces, empty "orphan Annie" nuclei, calcified spheres- excellent pr
ognosis, even when the adjacent lymph nodes are involved- can be caused by longterm radiotherapy to the neck, or RET-PTC fusion gene

Not for Resale & redistribution. Copyright joenny_king at www.ebay.com.

Q035.- List the four types of malignant thyroid tumors. Which has the best progn
osis and which has the worst?

A035.- papillary carcinoma - best- follicular carcinoma- medullary carcinoma- un


differentiated carcinoma -worst

Not for Resale & redistribution. Copyright joenny_king at www.ebay.com.


Q036.- what are the characteristics of medullary carcinoma of the thyroid?

A036.- Originates from C-cells in the thyroid, produces calcitonin.- histologica


lly: sheets of tumor cells in amyloid stroma- associated with multiple endocrine
neoplasia II, III

Not for Resale & redistribution. Copyright joenny_king at www.ebay.com.


Q037.- What are the causes of primary hyperparathyroidism? What are the laborato
ry findings associated with it? What are the clinical manifestations?

A037.- Most common cause is parathyroid adenoma. Primary parathyroid hyperplasia


and carcinoma are less likely. - PTH-like hormone can be produced by bronchogen
ic squamous cell carcinoma or renal cell carcinoma.- Lab findings include hyperc
alcemia/hypercalciuria, decreased serum phosphorus/increased urinary phosphorus,
increased serum alk.phos., and increased serum PTH.- Clinical symptoms include
cystic bone changes, "metastatic calcification", renal calculi, peptic ulcer.

Not for Resale & redistribution. Copyright joenny_king at www.ebay.com.


Q038.- What are the causes of secondary hyperparathyroidism? What are the lab va
lues and clinical characteristics?

A038.- Commonly caused by hypocalcemia in chronic renal disease. - Vitamin D con


version by the kidney is impaired, decreasing intestinal absorption of calcium.Lab values include decreased serum calcium, increased serum phosphorus, increas
ed serum alkaline phosphatase, increased serum PTH. - Clinical symptoms include
osteoclastic bone disease and metastatic calcification.- (Personal note: many ap
parent paradoxes in this disease)

Not for Resale & redistribution. Copyright joenny_king at www.ebay.com.


Q039.- What causes hypoparathyroidism? What are the symptoms?

A039.- The most common cause is accidental surgical excision during thyroidectom
y. - Sometimes associated with congenital thymic hypoplasia (DiGeorge syndrome).
- Resulting in severe hypocalcemia, manifested as increased neuromuscular excita
bility and tetany.

Not for Resale & redistribution. Copyright joenny_king at www.ebay.com.


Q040.- What is pseudo-hypoparathyroidism?

A040.- Multi-hormone resistance including PTH, and the pituitary hormones TSH, L
H, and FSH.- Clinical characteristics include unresponsiveness of the kidney to
PTH, shortened fourth and fifth metacarpal and metatarsals, short stature, and o
ther skeletal abnormalities.

Not for Resale & redistribution. Copyright joenny_king at www.ebay.com.


Q041.- What are the causes of Cushing's syndrome? What is the difference between
Cushing's syndrome and Cushing's disease?

A041.- Exogenous corticosteroid medication- Hyper production of ACTH by pituitar


y adenoma- Adrenal cortical adenoma or carcinoma- Ectopic production of ACTH (by
small cell carcinoma of the lung)- Cushing's syndrome results from increased ci
rculating cortisol from any source, Cushing's disease refers to only hyper produ
ction of ACTH by pituitary adenoma.

Not for Resale & redistribution. Copyright joenny_king at www.ebay.com.


Q042.- What is primary hyperaldosteronism?

A042.- Conn syndrome, usually due to an aldosterone producing adrenocortical ade


noma or carcinoma.- Causes hypertension, sodium and water retention, and hypokal
emia.- Decreased serum renin is present due to negative feedback from hypertensi
on.

Not for Resale & redistribution. Copyright joenny_king at www.ebay.com.


Q043.- What is secondary hyperaldosteronism?

A043.- Caused by stimulation of the renin angiotensin system. - Serum renin is i


ncreased in contrast to primary hyperaldosteronism.- Usually secondary to renal
ischemia, renal tumors, and edema (cirrhosis, nephrotic syndrome, cardiac failur
e).

Not for Resale & redistribution. Copyright joenny_king at www.ebay.com.


Q044.- What is adrenal virilism?

A044.- Congenital enzyme defects that lead to diminished cortisol production and
compensatory increased ACTH. - This leads to adrenal hyperplasia and androgenic
steroid production.- Deficiencies include 21-hydroxylase deficiency and 11-hydr
oxylase deficiency.- Clinical characteristics include precocious puberty in male
s and virilism in females.

Not for Resale & redistribution. Copyright joenny_king at www.ebay.com.


Q045.- What is Addison disease? Symptoms and labs?

A045.- Primary adrenocortical deficiency, usually due to idiopathic adrenal atro


phy. - Can also be caused by tuberculosis, metastatic tumor, and infection. - Ch
aracteristics include hypotension, skin pigmentation, low serum sodium, chloride
, glucose, bicarbonate and increased potassium.

Not for Resale & redistribution. Copyright joenny_king at www.ebay.com.


Q046.- What is Waterhouse-Friedrichsen syndrome?

A046.- Catastrophic adrenal insufficiency and vascular collapse due to hemorrhag


ic necrosis of the adrenal cortex. - Often associated with DIC and characteristi
cally due to meningococcal meningitis (Neisseria meningitis).

Not for Resale & redistribution. Copyright joenny_king at www.ebay.com.


Q047.- What is a pheochromocytoma?

A047.- Adrenal medulla tumor of chromaffin cells. - Causes paroxysmal hypertensi


on due to hyperproduction of catecholamines. - Increased urinary excretion of ca
techolamines and metabolites (metanephrine, normetanephrine, vanillylmandelic ac
id). - Can also cause hyperglycemia.

Not for Resale & redistribution. Copyright joenny_king at www.ebay.com.


Q048.- What is a neuroblastoma?

A048.- A highly malignant catecholamine producing tumor occurring in early child


hood. - Urinary catecholamines and metabolites are seen as in pheochromocytoma.
- Hypertension is common. - Originates in the adrenal medulla and presents as a
large abdominal mass.

Not for Resale & redistribution. Copyright joenny_king at www.ebay.com.


Q049.- Epidemiology and etiological factors of type 1 diabetes mellitus?

A049.- Usually early in life, less common than type 2 disease. - The disease is
due to failure of insulin synthesis by beta cells of the pancreatic islets. - Ca
use may be due to genetic predisposition or autoimmune inflammation of the islet
s. - Incidences significantly higher in individuals with a mutation in HLA DQ, a
nd in HLA DR3/HLA DR4 positive individuals.

Not for Resale & redistribution. Copyright joenny_king at www.ebay.com.


Q050.- Clinical characteristics of type 1 diabetes?

A050.- Without insulin replacement: carbohydrate intolerance with hyperglycemia


leading to polyuria, polydipsia, weight loss, ketoacidosis, coma, and death. - K
etoacidosis results from increased metabolism of fat leading to production of "k

etone bodies".

Q051.- Epidemiology and etiological factors of type 2 diabetes mellitus?- Causes


of secondary type 2 diabetes?

A051.- Type 2 is much more common than type 1, and tends to begin in middle age.
Positive family history is often present. - Type 2 disease is due to increased
insulin resistance, and is associated with obesity.- Type 2 can also occur as a
secondary phenomenon in certain conditions including:- pancreatic disease -- hem
ochromatosis, pancreatitis, pancreas carcinoma- other endocrine -- Cushing's, ac
romegaly, glucagon hypersecretion, pheochromocytoma, hyperthyroidism- pregnancy
-- gestational diabetes, hyperglycemic mother leads to infants with hyperplastic
islets

Q052.- Anatomic changes in diabetes mellitus?

A052.- Kidney: widened glomerular basement membrane, glomerulosclerosis, exudati


ve lesions, pyelonephritis.- Cardiovascular: atherosclerosis, MI, peripheral vas
cular disease, capillary basement membrane thickening- Eye: cataract formation,
proliferative retinopathy (exudates, edema, hemorrhage, micro aneurysms)- Nervou
s system: peripheral neuropathy- Liver: fatty changes- Skin: xanthomas, infectio
ns

Q053.- What is an insulinoma?

A053.- A beta cell tumor, the most common islet tumor. - Characterized by insuli
n secretion with high levels of C-peptide. - Symptoms include Whipple's triad: p
aroxysmal hypoglycemia, CNS dysfunction (confusion, anxiety, coma, convulsions),
immediate recovery with glucose administration

Q054.- What is a gastrinoma?

A054.- Gastrin secreting tumor, often malignant and sometimes found outside the
pancreas.- Causes gastrin hypersecretion and hypergastrinemia.- Causes Zollinger
-Ellison syndrome (high gastric HCl, PUD, hypergastrinemia)

Q055.- What is a glucagonoma?

A055.- A glucagon secreting alpha cell tumor that causes secondary diabetes mell
itus. - Also causes a skin lesion called necrolytic migratory erythema in 80% of
cases, a spread of erythematous blisters and swelling across areas subject to g
reater friction and pressure, including the lower abdomen, buttocks, perineum, a
nd groin.

Q056.- What is a VIPoma?

A056.- Vasoactive intestinal peptide secreting tumor. - Causes watery diarrhea,


hypokalemia, and achlorhydria syndrome (pancreatic cholera).

Q057.- What is MEN-1?

A057.- Werner syndrome: hyperplasia or tumors of the pituitary, parathyroid, and


pancreatic islets (3 Ps). - May also involve the thyroid and adrenal cortex. Li
nked to mutations in the MEN-1 gene (chr 11).- Pancreatic component includes ZE
syndrome, insulinoma, or pancreatic cholera.- autosomal dominant

Q058.- What is MEN-2a?

A058.- Sipple syndrome, includes pheochromocytoma, medullary carcinoma of the th


yroid, and hyperparathyroidism due to hyperplasia or tumor. - Due to mutation in
the ret oncogene on chromosome 10.- autosomal dominant

Q059.- What is MEN-2b (MEN-3)?

A059.- Characterized by pheochromocytoma, medullary carcinoma, and multiple muco


cutaneous neuromas. - Unlike MEN-2a no hyperparathyroidism. - Linked to mutation

s in the ret oncogene on chromosome 10.- autosomal dominant

Q060.- Diabetes Type 1

A060.- Insulin Dependent

Q061.- Diabetes Type 2

A061.- Insulin Independent- Not ketosis prone- Children with sedentary life- Str
ong genetic predisposition- Controlled mainly with oral agents

Q062.- Goals of Diabetes Treatment

A062.- Achieve and maintain normal blood glucose levels- BG monitoring- Drug the
rapy- Diet - Exercise- Rest and reduce stress

Q063.- Patient Teaching

A063.- What is the patient's perspective

Q064.- Diabetes Diagnostic Studies

A064.- Blood glucose: - *fasting- *random- *ac or pc- Glycosylated hemoglobin:(A


1C or HbA1c) normal <6- *over past three months- Oral glucose tolerance test- Ur
ine testing for ketones especially for sick days

Q065.- Other events indicating BG monitoring

A065.- Change in condition that resembles hypo/hyperglycemia- such as stroke- Us


e of TPN

Q066.- S/S of Hypoglycemia

A066.- Weakness, fatigue- Headache, dizziness- Sweating- Anxiety, tremors- Lack


of cordination- Hunger, nausea- Impaired vision- LOC

Q067.- S/S of Hyperglycemia

A067.- Thirst- Frequent urination- Hunger- Blurred vision- Drowsiness- Nausea- D


ry skin- Hyperosmolar blood serum

Q068.- Complications of Hyperglycemia

A068.- Damaged blood vessels, nerves and decreased ability to fight infectionsAffects kidneys, eyes, skin, feet, heart

Q069.- Classifications of DM

A069.- Type 1 - Type 2- Gestational Diabetes- Diabetes due to drugs that raise B
G- Chronic pancreatitis- Removal of pancrease- Cushings disease- Acromegaly- Phe
ochromocytoma (tumor in adrenal gland)

Q070.- Defects of Type 2 DM

A070.- Decreased Tissue sensitivity to insulin (dysfunction at the receptor site


)- Beta cells: less repsonsive or decrease insulin production- Hepatic Insensiti
vity: liver continues to release glucose despite high insulin levels

Q071.- Differential Diagnosis

A071.- Young/old- Thin/obese- Ketone/non-ketotic- Abrupt/progressive onset- C-pe


ptide: low in type 1, normal or elevated in type 2

Q072.- Clinical Findings Type 1

A072.- Polyuria- Polyphagia- Polydipsia (increased thirst)- Rapid weight loss- W


eakness- Thin- Young- Ketones

Q073.- Clinical Findings Type 2

A073.- Polyuria- Polyphagia- Polydipsia- Fatigue- Blurred vision- Older- ObeseNo obvious signs, found at routine MD visit

Q074.- Exercise Induced Hypoglycemia

A074.- Can occur up to 24 hours after exercising- Test before exercising and eat
a snack if necessary- Eat good diet after

Q075.- Sulfonylureas

A075.- Mechanism of action: increase insulin secretion by beta cells- Timing: gi


ve AC, QD-BID, duration 12-14 hours- Medications:- *Glipizide/Glucotrol- Risks:
hypoglycemia

Q076.- Meglitinides

A076.- Mechanism of action: increase insulin secretion by beta cells- Timing: gi


ve AC, QD-TID, duration unknown- Medications:- *Repaglinide/prandine- *StarlixRisks: hypoglycemia (less than sulfonylureas

Q077.- Biguanides

A077.- Mechanism of action: Decrease hepatic glucose production (gluconeogenesis


)- Timing: Usually BID (with food), onset: 24-48hrs duration 12-14hrs- Medicatio
ns:- *Metformin: Glucophage- Nursing considerations: titrate dose upward over 12 weeks, no risk of hypoglycemia- Risks: lactic acidosis, stop med with acute il
lness, renal failure, CHF, and dyes

Q078.- Thiazolidinediones (TZDs)

A078.- Mechanism of action: Increase tissue sensitivity to insulin - Timing: QDBID, duration: long half life, can cause liver toxicity- Medications:- *Pioglita
zone: Actos (major ankle swelling)- *Rosiglitazone: Avandia- *Regal- Risks: Weig
ht gain, edema, several weeks to be effective

Q079.- Drawing up mixed doses

A079.- Inject air into long acting insulin (cloudy)- Inject air into the short a
cting insulin (clear)- Draw up short acting insulin- Draw up long acting insulin
(mixing vile first)- Make sure to have two licensed RNs confirm both draws of i
nsulin

Q080.- Acute Complications of Diabetes

A080.- Type 1: Diabetic Ketoacidosis (DKA)- Type 2: Hyperglycemic hyperosmolar n


onketotic syndrome (HHNKS)- Hypoglycemia/insulin shock

Q081.- Diabetic Nephropathy

A081.- Damage to the renal vessels can lead to kidney failure- 1/3 of new ESRD p
atients are diabetic- Leading cause of kidney failure

Q082.- Foot problems

A082.- 60-80% of amputations are DM patients - Causes: infections, non-healing u


lcers- Increased risk due to: neuropathy, poor circulation, high BG levels impai
r WBC function

Q083.- NIDDM develop HHNKS

A083.- Hyperglycemic Hyperosmolar Nonketotic Syndrome or Coma for Type 2- some i


nsulin is available, therefore ketones do not build up in the serum- hyperosmola
r state leads to diuresis and decrease in LOC

Q084.- HHNKS: Physiology

A084.- Sever osmotic diuresis loss of water, - glucose and electrolytes causes d
ehydration and high serum Na, - decreased intervascular volume, - all leading to
more stress thus causing more glucose to enter the blood stream

Q085.- HHNKS: Causes

A085.- Type 2 DM, - precipitated by illness, - inability to drink fluids at will

Q086.- HHNKS: S/S

A086.- Hyperglycemia, - dehydration (HR up, BP down), - neuro sx, - coma (no ket
osis, acidosis as some insulin present)

Q087.- HHNKS: Treatment

A087.- Rehydrate- Correct electrolyte imbalance (Potassium usually needed)- Norm


alize BG- Treat underlying cause

Q088.- DKA Cause

A088.- DM Type 1- Failure to take insulin, not enough insulin, infection, stress
- Potassium is outside the cells

Q089.- DKA Physiology

A089.- Lypolysis, - fatty acids converted to energy by liver (Ketones byproduct)


, - acid pH, - cellular dehydration

Q090.- DKA S/S

A090.- BG > 300-800- 3Ps- Weak, lethargic- Kussmaul resp- N/V, anorexia- Headach
e- Abdominal pain- Fruit breath

Q091.- DKA Treatment

A091.- Normalize BG (regulare insulin given IV)- Replace fluids- Replace Potassi
um (unless hyperkalemia or no urine output)

Q092.- Intravenous Regular Insulin

A092.- Keep it continuous and monitor BG often- Only use Regular insulin- If ir
is add to IV bag, it can adhere to tubing, so flush insulin through new tubing

Q093.- Adrenal Glands

A093.- Adrenal Cortex- Adrenal Medulla

Q094.- Adrenal Cortex

A094.- Glucocorticoids: cortisol- Mineralcorticoids: aidosterone- Sex Hormones

Q095.- Adrenal Medulla

A095.- Epinephrine- Norepinephrine

Q096.- Addison's Disease

A096.- Atrophy of adrenal cortex- Hyposecretion of cortisol- Anterior pituitary


increases ACTH- Causing increase excretion of Na and H20 (dehydration)- Hypotens
ion, weak & irregular pulse, weight loss, anemia, hypoglycemia, mood changes, in
ability to handle stress, dark pigmentation of mucous membranes

Q097.- Addison's Disease Treatment

A097.- Lifelong cortisol replacement therapy- Dose needs to be increased for phy
sical and emotional stessors- High Na intake

Q098.- Addison's Crisis

A098.- Sudden life-threatening exacerbation triggered by a stressor- Low Na, low


BG, high K, dehydration, extremely low BP, tachycardia- Treatment: IV cortisol,
Aggressive IV NS fluid, glucose admin, B/P meds and Na retention

Q099.- Cushing's Syndrome

A099.- Excessive secretion of corticosteroids, hyperplasia- S/S: muscle wasting,


weakness, protein depletion, *moonface, buffalo hump and trunk obesity*, Na and
H20 retention, *decreased inflammatory response*- Treatment: *Medications to su
press adrenal glands: Lysodrem, Cytadren*

Q100.- Administration of Steroids

A100.- May raise Na and lower K- Increase fluid volume- Indications: suppress in
flammatory and immune response (COPD, arthritis, lupus)- Treat addison's - Treat
allergic reactions

Q101.- Precautions for Steroid Use

A101.- Increase BG levels- Elderly: raises Na and BP, lowers K- Always give (ora
lly) with food(raises HCL production in stomach)- Never abruptly D/C: need to ta
per off (check meds in patients transferred from ICU or CCU)- Wear Medic-alert I
D bracelet

Q102.- Side Effects of Steroids

A102.- PVD- Steroid induced DM- Decreased immune response- Inflammation- Osteopo
rosis- HTN, edema- Buffalo hump & moon face- Acne- Bruising- Mood changes- Hirsu
tism (extra hair)- amenorrhea (no period)

Q103.- Mineralcorticoids: Hyperaldosteronism

A103.- Increase reabsorption of Na and H2O- Hypernatremia (maybe edema)- Leads t


o increased urinary output- Leads to *hypokalemia* causing muscle weakness, cram
ping, fatigue, tetany, and paresthesia (alkalosis with loss of H+)- Women 7x mor
e than men

Q104.- Adrenal Medulla Disorder: Pheochromocytoma

A104.- Tumor %80 benign/Malifnant 10%- Tumors located in other sites of adrenal
tissue (aorta, ovaries, spleen)- Genetic predisposition- Secretes excessive cate
cholemines (epi and nor)

Q105.- Physiology of Pheochromocytoma

A105.- Flight or Fight - Increase HR- HTN- Na and fluid retention- Increased glu
cose during stress

Q106.- S/S of Pheochromocytoma

A106.- HTN (220/140)- Tachycardia, palpitations- Headache- Tremors- Hyperhidrosi


s (excessive sweating)- Dx: 24 hour urine to test for catecholamines

Q107.- Posterior Pituitary

A107.- ADH or vasopressin- Oxytocin

Q108.- Vasopressin

A108.- Regulates Reabsorption of H2O in kidneys

Q109.- Hypersecretion of Vasopressin- SIADH

A109.- Syndrome of inappropriate antiduretic Hormone secretion- Excessive ADH pr


oduction- Causes: oat cell lung cancer, CNS infection, tumor,- head trauma- anyt
hing that lowers cardiac output stimulates ADH production- Serum hyponatremia- I
>O- *Interstitial edema (puffy eyes, hands, face)- Tx: Fluid restriction, diuret
ic, replace K, albumin (osmotic diuretic)

Q110.- Hyposecretion of Vasopressin- Diabetes Insipidus

A110.- Lack of ADH- Increased urine output (polyuria) - Increased thirst- Dilute
urine- Urine Specific Gravity < 1.005- Tx: nasal spray: desmopresin

Q111.- Anterior Pituitary Gland

A111.- ACTH: acts on the adrenals- TSH: acts on the thyroid- FSH: acts on the ov
aries and testes- GH: Somatotropin

Q112.- Growht Hormone

A112.- Somatotropin- Hypersecretion: giantism/acromegaly- Hyposecretion: Dwarfis


m/Simmonds

Q113.- Dwarfism

A113.- Hyposecretion of GH by the ant pituitary gland- Causes: inherited, tumors


, surgical removal, idiopathic- S/S: slowed growth of skeletal and soft tissues
during childhood- Tx: growth hormone (only effective in children)

Q114.- Simmond's

A114.- Causes: hypopituitarism: total absence of all pituitary secretions - Rare

Q115.- Gigantism

A115.- Hypersecretion of GH by the ant pituitary gland- Excessive amounts of GH


during childhood

Q116.- Acromegaly

A116.- Excess of GH in adults- Hyperplasia - Huge lower jaw- Headache- Increased


pressure in head that can lead to blindness d/t pressure on the optic nerve

Q117.- Thyroid

A117.- T3 & T4 secretion is controlled by TSH from the ant pit.- T3 and T4 regul

ate the metabolism rate

Q118.- Grave's Disease

A118.- Excessive output of Thyroid hormones by stimulation of the thyroid gland


by immunoglobins- S/S: nervousness, tachycardia, bulging eyes (exephtalmos), inc
reased appetite, increased weight loss, muscle weakness and fatigue, increased B
P, A-fib, goiter- Tx: - *irradiation by iodine isotope- *medications that interf
ere with hormone production sa:- propythioracil (Propacil)- thimazole (Tapazole)
- Methimazole,- PTU

Q119.- Thyroid Storm

A119.- Thyrotoxicosis

Q120.- Hypothyroidism

A120.- Insufficient thyroid hormone due to: - *Primary: thyroid dysfunction (low
T3 & T4, high TSH0- *Secondary: anterior pituitary / hypothalmic dysfunction (l
ow T3 & T4, low TSH)- Causes: tx for hyperthyroidism- S/S: - *weakness and letha
rgy- *slowed speech, thick tongue- *flat affect- *memory loss- *decreased heart
rate- *poor appetite- *intolerance to cold - hypothermia- *dry skin, brittle nai
ls- *weight gain- *myxedema- Tx: leve-thyroxine = synthroid, Levoxyl

Q121.- Myxedema Coma

A121.- Severe, end-stage hypothyroidism- Tx: maintain airways, meds, corticoster


oids (for suppressed adrenal gland), correct BG and electrolyte levels, conserve
r body heat

Q122.- Special Considerations for HRT

A122.- May takes weeks to feel better- Take medications daily- Tx can speed up b
ody functions causing problems for patients with:- *Corinary Artery Disease- *DM
- *Adrenal insufficiency

Q123.- Radioactive Iodine

A123.- Destruction of all or part of the gland- Given orally 3-5mos for effectMay cause hypothyroidism- Can damage parathyroid

Q124.- Tyroidetomy

A124.- Post-op care: monitor for airway obstruction, sauna mask- Keep intubation
/tracheostomy equip at bedside

Q125.- S/S Thyroid Storm

A125.- High temp 102-106- Severe tachycardia, high CHF hypoxia- Manic/psychotic
behavior- Extreme diaphoresis, vomiting, diarrhea- Coma

Q126.- Endemic Goiters

A126.- Non-toxic- Iodine deficiency- Defect in thyroid hormone synthesis of T3 &


T4- Increases TSH which causes hyperplasia of gland- Firm or soft, enlarged- Ma
y have difficulty swallowing - Can progress to toxic goiter- Tx: may shrink with
Thyroid hormone replacement therapy

Q127.- Parathyroid Hormone

A127.- Acts on bone, intestine, kidney, to regulate calcium and phosphate metabo
lism- Fosters normal excitability of nerves and muscles

Q128.- Hypoparathyroidism

A128.- Equals hypocalcemia- Causes: accidental removal with thyroidectomy, Surge


ry or radiation of throat or head, Autoimmune destruction- Dx: low calcium, high
phosphorus

Q129.- S/S Hypoparathyroidism

A129.- Same as hypocalcemia- Paresthesia (tingling fingers)- Tetany/neuromuscula


r irritability- *+ trousseau's sign (upper arm compression spasm)- *+ chovstek's
sign (tap on facial nerve produces twitch)- *laryngeal spasm (crowing respirati
ons)- EKG changes- Digitalis resistance

Q130.- Tx Hypoparathyroidism

A130.- Sever Case:- IV Calcium gluconate (not given IM, can damage tissue)- Mild
Case:- PO calcium and vitamin D - Amphogel (binds phosphorus)

Q131.- Hyperparathyroidism

A131.- Primary: Excessive secretion of PTH by para- Secondary: seen in patients


with CRF due to retention of phosphorus causing the para to increase stimulation
of PTH

Q132.- S/S Hyperparathyroidism

A132.- Hypercalcemia- Muscle weakness- Constipation- Osteoporosis- Arrhythmias

Q133.- Diabetes Mellitus

A133.- Chronic metabolic disorder characterized by high blood sugar (hyperglycem


ia = > 126 mg/dl) - Affects CHO, Fat & Protein Metabolism- Damages: - Blood Ves
sels (Heart Disease, Stroke, HTN, PAD);* - Nerves (neuropathy);* - Kidney (nep
hropathy);* - Eyes* (retinopathy)- Caused by Insulin Deficiency, Insulin Resist
ance Or both

Q134.- Incidence of DM

A134.- Affects all age groups- Affects 18.2 million people in US

Q135.- Glycogenesis

A135.- Liver converts glucose to glycogen- Glycogen stored in liver and muscle c
ells

Q136.- Glycogenolysis

A136.- Liver converts glycogen to glucose

Q137.- Gluconeogenesis

A137.- Liver Synthesizes glucose from non-carbohydrate sources: - Amino acids, l


actic acid, glycerol

Q138.- Glucose

A138.- Primary energy source, provides 4kcal/gram

Q139.- fat metabolism- 9kcal/gram

A139.- CHO to triglycerides (via liver) -> fatty acids (via lipase) -> Ketones (
via liver) this occurs when large amounts of fatty acids are used for energy sou
rce, causes ketoacidosis/ metabolic acidosis

Q140.- Protein Metabolism- 4kcal/gram

A140.- excess AA converted to FA ketones, or glucose. - Body breaks down protein


s and uses the amino acids as a major substrate for GLUCONEOGENESIS

Q141.- Kidney Regulation of Glucose

A141.- Glucose almost totally reabsorbed by renal tubules - However, when plasma
glucose is > 180 mg/dl, glucose spills in urine (glycosuria)

Q142.- Brain & NS and Glucose

A142.- Can only use glucose as energy source- Only stores enough glucose to last
few minutes- Prolonged & severe hypoglycemia can result in brain death

Q143.- Muscle tissue and glucose

A143.- Uses either glucose or fatty acids - At rest -> fatty acids- Exercise ->
glucose- Muscle fibers more permeable to glucose during exercise due to contract
ion process even in absence of insulin.

Q144.- Alpha Cells of Pancreas

A144.- Secretes Glucagon: BS- Glucagon: Regulated by blood sugar level - BS= Glu
cagon = BS- Maintains BS between meals & fasting/starvation states- BS by hepati
c glucose production

Q145.- Beta Cells of Pancreas

A145.- Secretes Insulin= BS- Insulin: Regulated by blood sugar level- BS = Insul
in = BS- Insulin receptors on cells of the body recognize insulin and allow gluc
ose to be transported into the cell where glucose is converted to energy

Q146.- Action of Insulin

A146.- Promotes glucose uptake by target cells, - Promotes storage of excess glu
cose in fat cells, - Increases protein synthesis by increasing transport of amin
o acids into body cells- Inhibits protein breakdown

Q147.- Glucagon

A147.- BS - Maintains BS between meals & fasting-starvation states- Hepatic gluc


ose production

Q148.- Catecholamines

A148.- Epinephrine & Norepinephrine (adrenal medulla)- BS during stress- Promote

s conversion of glycogen to glucose (Glycogenolysis)

Q149.- Glucocorticoid

A149.- Cortisol (adrenal cortex)- BS during stress- Hepatic glucose production

Q150.- Growth Hormone

A150.- Secreted by anterior pituitary- Promotes protein synthesis in all cells,


increases fat mobilization and use of fatty acids for energy and decrease use of
CHO

Q151.- Diabetes Mellitus Type I- (juvenile onset, insulin dependent)

A151.- accounts for 10% of cases, occurs at any age, Genetics and autoimmune res
ponses plays a role,

Q152.- S/S DM Type 1

A152.- thin, Peak age = 12 (althoughSudden onset,Body type may occur any age)- F
atigue / Listlessness, Polyuria, Polydipsia, Polyphagia, Weight Loss, Glucosuria
, Ketoacidosis

Q153.- Diabetes Mellitus Type II- (adult onset, non-insulin dependent)

A153.- Accounts for 90% of all cases of diabetes characterized by - Insulin Resi
stance - Insulin Secretion- Hepatic Glucose Production

Q154.- DM I etiology

A154.- Genetics: associated w/ Human Leukocyte antigen- Hypothetical Environment


al Antigen: triggers autoimmune response, autoimmune response: destroys Beta cel
ls

Q155.- Etiology of DM II

A155.- Insulin Resistance:- Fewer insulin receptor sites on* cell membrane- Insu
lin receptor abnormality*- Abnormality in glucose* transporter- Function only af
ter insulin binds to receptor site- Insulin Secretion: - Hyprinsulinemia leads t
o beta cell exhaustion- Beta cells poop out- Cells unable Hepatic Glucoseroductio
n: Insulin Deficiency to use glucose as energy source - Cells starving- Liver c
onverts glycogen to glucose - Worsens hyperglycemia

Q156.- DM II Risk Factors

A156.- Age >20,- obesity (85% obese), - Genetics (20-50%.. greater than type 1),
- Race (Black, Native Americans, Hispanics) - Impaired Fasting Glucose (pre dia
betes 110-125mg/dl), - Gestational Diabetes (hyperglycemia in pregnancy 5-10yrs
later), - Insulin Resistance Syndrome (Metabolic Syndrome, Syndrome X, Presence
of Four atherosclerotic risk factors: Insulin resistance, Hypertension, Hyperlip
idemia, Central obesity)

Q157.- MODY (maturity onset diabetes of the young)

A157.- DM II onset age <25yo,- Often Obese- Genetic Defect ==> Autosomal Dominan
t

Q158.- S/S DM II

A158.- Asymptomatic,- > 40, Obese, 3 P's: (Polyuria, Polydipsia, Polyphagia), Sk

in infections, Vaginal Yeast Infections, Cystitis- NO WEIGHT LOSS- NO KETOACIDOS


IS

Q159.- FPG: Fasting Plasma Glucose

A159.- Normal; 70-100mg/dl, - IFG Pre diabetes: 110-125 mg/dl, - DM:> 126 mg/dl confirmed with 2 FPG

Q160.- Random Blood Sugar

A160.- Non fasting test- DM= > 200 mg/dl PLUS the presence of classic sx

Q161.- OGTT: - Oral Glucose Tolerance Test

A161.- Used to detect gestational DM or to detect DM-2 in person who has a norma
l FPG- DM= > 200 mg/dl after 2 hours

Q162.- Hemoglobin AIC

A162.- Normal = 4-6%- Goal for DM = <7%- Determines how well DM is being managed
- Reflects average glucose for the preceding 6-12 week period- Not affected by f
ood intake, stress or exercise

Q163.- DKA: Diabetic Ketoacidosis

A163.- acute complication in DM-1- When Does it Occur?- Onset of disease (before
dx)*- Omit insulin or insufficient* dose- Illness or Physical-Emotional Stress
==*> BS 2nd to Counter Regulatory Hormones

Q164.- Diabetes Lab findings

A164.- pH (< 7.35)- BS (often 600-700)- HCO3 (< 15 mE/L)- Ketones in blood and u
rine- Glucose in urine- Compensate = Kussmauls respirations

Q165.- HHNC:- Hyperglycemic Hyperosmolar Nonketotic Coma

A165.- Acute complication of DM 2- Lab Findings- BS > 600 mg/dl- Serum osmolarit
y > 310 mOs/L- No ketoacidosis

Q166.- Hypoglycemia or Insulin Reactions

A166.- Occurs in DM-1 or DM-2- BS = < 50 mg/dl

Q167.- S/S Hypoglycemia

A167.- Cerebral Function, Autonomic NS Activity; conscience-> injest sugar, unco


nscience-> HOSPITALIZE

Q168.- Somogyi Phenomenon

A168.- Occurs with DM-1- Morning hyperglycemia 2nd nocturnal hypoglycemia- Noctu
rnal hypoglycemia - Occurs because evening dose of* insulin too high- Morning Hy
perglycemia = Compensatory response- counter* regulatory hormones - Hepatic gluc
ose production- Confirm Somogyi effect by measuring BS 2-3 am- Tx: Decrease even
ing insulin dose

Q169.- Dawn Phenomenon

A169.- Occurs in DM-1 or DM-2- Morning hyperglycemia without nocturnal hypoglyce


mia- Caused by circadian rhythm secretion of growth hormone- Tx: Increase PM ins
ulin dose - Switch evening NPH to HS

Q170.- Retinopathy

A170.- Chronic complication of DM- microvascular disorder- all DM1, 60% DM 2- da


mage/ruptured blood vessels in eyes

Q171.- Nephropathy

A171.- Chronic complication of DM- microvascular disorder- DM-1: present after 1


5 years- DM-2: present at time of dx- Affects blood vessels that supply glomerul
i - Leading cause of ESRD- First sign of diabetic nephropathy is an urinary albu
min excretion (microalbuminuria)- Microalbuminuria defined as urine protein loss
between 30-300 mg/day.

Q172.- Neuropathy

A172.- Chronic complication of DM- microvascular disorder- Endothelial thickenin


g of blood vessels that supply nerve -> nerve ischemia- Demyelinization of Schwa
nn Cells which slows nerve conduction - Peripheral Neuropathy Most common- Affe
cts lower extremities - Numbness / Tingling- Loss of Sensation

Q173.- Chronic Complications of DM-- Macrovascular Disorders

A173.- DM-2: present at time of dx - DM-1: onset depends on age & duration of DM
- CAD (agina, MI), CVA(stroke), PAD (pain/intermittent claudication)

Q174.- Thyroid function

A174.- Takes iodine and converts it to thyroid hormones - T3 & T4- Iodine Necess
ary for synthesis of thyroid hormones

Q175.- Follicular Cells

A175.- Functional unit of thyroid gland; - Tiny sac-like structures that absorb
iodine; - Combine iodine and tyrosine (amino acid) to make T3 and T4.- Have appr
oximately 2-3 months supply of thyroid hormone stored in gland

Q176.- Hypothyroid

A176.- Insufficient thyroid hormone- BMR- Slows down all body activity- Classifi
cation- Congenital- Acquired

Q177.- Congenital Hypothyroid

A177.- Affects approx 1 of 4000 births - Caused by- Absent thyroid gland- Inadeq
uate hormone production- Deficient TSH- Cretinism=manifestations of untx congeni
tal hypothyroidism- Growth retardation- Mental retardation- Tx: Hormone replacem
ent

Q178.- Acquired Hypothyroid

A178.- Primary Hypothyroid = Thyroid Gland Dysfunction- Secondary Hypothyroid =


Hypothalmic or Pituitary Dysfunction (less common)

Q179.- Causes of Hypothyroidism

A179.- Hashimotos Thyroiditis (most common)- Surgical-induced hypothyroidism- Rad


iation-induced hypothyroidism

Q180.- Hashimoto s Thyroiditis

A180.- Most common- Primarily affects women (5:1 ratio)- form autoantibodies whi
ch react with t Autoimmune disorder hyroid destroys glandantigens - May produce
transient hyperthyroid state = leakage of preformed thyroid hormone from damaged
cells

Q181.- Surgical Induced Hypothyroid

A181.- Thyroidectomy- Complete or partial removal of thyroid gland

Q182.- Radioactive Iodine (RAI)

A182.- Used to tx hyperthyroid- Thyroid gland concentrates RAI- become hypothyr


oid- Destroys gland

Q183.- S/S Hypothyroidism

A183.- Sx Vary ==> May be subtle or overt- Gradual onset, weakness and fatigue,
skin dry and course- Hair course and brittle, Accumulation of hydrophilic mucopo
lysaccharide substance, Puffy appearance especially around eyes, Bradycardia, De
creased RR, Decreased appetite, Menorrhagia, Easy fatigueabiity, Physically/ment
ally sluggish, Severely hypothyroid, unable to metabolizes sedatives, analgesics
and anesthetic drugs, T3, T4, TSH, Cholesterol

Q184.- Hyperthyroidism

A184.- Excess thyroid hormones- BMR- Accelerates all body activity

Q185.- Primary Hyperthyroid

A185.- Thyroid gland dysfunction

Q186.- Secondary Hyperthyroid

A186.- Hypothalamus or Pituitary Dysfunction

Q187.- Hyperthyroid Causes

A187.- Graves Disease (most common cause)- other causes not discussed

Q188.- S/S Grave s Disease

A188.- Presentation may be subtle or dramatic ==> Depends on hormone level, Onse
t: Insidious, heat intolerance, smooth velvety skin, thin fine silky hair, break
able nails, Graves Dermopathy (Rare occurs after many years), Exophthalmos (occu
rs in 1/3 pts), Goiter (2-3x normal size), Increased CO, Tachycardia, Angina, SO
B with exertion, increased appetite, Amenorrhea, muscle protein breakdown= used
as fuel, Restless, anxious, nervous, insomnia, difficulty concentrating, Increas
ed sensitivity to catecholamines (epi/norepi)- T3, T4- TSH- Cholesterol

Q189.- Chatecolamines

A189.- Epi and Norepi. Hormones of Adrenal Medulla. - Increase BP, HR, Respirati

ons- Increases peripheral vasoconstriction- Increases sweating

Q190.- Aldosterone

A190.- Hormone of Adrenal Cortex:- Regulates Na+, K+, & Water levels

Q191.- Sex Hormone

A191.- Adrenal cortex produces small amounts of sex hormone in both males and fe
males

Q192.- Causes of Cushing s Syndrome

A192.- Pituitary Tumor -> Increased ACTH- Adrenal Tumor -> Increased Cortisol- L
ung Cancer -> Increased ACTH- Long term prednisone therapy -> Suppresses ACTH

Q193.- S/S of Cushing s Syndrome

A193.- Wide range of emotions, - Moon face (full/round), - Overt DM develops in


approx 20% of patients, - Muscle Wasting esp. in extremities, - Decreased humora
l and cell mediated immunity, - Mask infection/inflammation, - (Increased Androg
en) Acne, Facial hair (hirsutism), - Osteoporosis, - (loss of collagen) Produces
thin/weak integumentary tissue, - Peptic ulcers, - Increased Na/water retention
-> Hypertension, edema, Increased ACTH -> melenocyte stimulating activity

Q194.- Primary Adrenal Insufficiency

A194.- Cortisol, Aldosterone, ACTH- Relatively rare- 5 cases / 100,000 persons- F


emale:Male = 2:1 ratio- Autoimmunity:- (Destruction of adrenal gland- Makes up a
bout 80% of Cases)- Other:- Metastatic destruction of adrenal gland- Tuberculosi

s - common in countries where TB is prevalent- Adrenal hemorrhage (anticoagulant


s/trauma)- Adrenal infarction (thrombosis)

Q195.- S/S Primary Adrenal Insufficiency

A195.- Gradual onset, Sx appear when 90% of gland destroyed, ( Cortisol) Hypoglyc
emia- Poor tolerance to stress, infection, trauma, ( Aldosterone): Excrete Na/Wat
er; Retain K+, Hypovolemia, Hyponatremia, Hyperkalemia, Hypotension, Weight loss
- ACTH: Melenocyte stimulating activity- Androgen: Females = sparse axillary/pub
ic hair

Q196.- common symptoms of hyperthyroidism?

A196.- nervousness, - anxiety, - irritability, - insomnia, - heat intolerance, sweating, - palpitations, - weight loss with increased appetite, - fatigue, - w
eakness, - emotional lability, - diarrhea

Q197.- common signs of hyperthyroidism?

A197.- enlarged thyroid gland, - warm skin, - thyroid stare , - lid lag, - exop
hthalmos, - proptosis, - tremor, - tachycardia, - AFib

Q198.- what disease presents in part with ophthalmoplegia and pretibial myxedema
?

A198.- Graves disease

Q199.- what endocrine check should you do when patient present with new-onset AF
ib?

A199.- TSH level

Q200.- what is struma ovarii?

A200.- an ovarian teratoma that secretes thyroid hormone

Q201.- most common cause of hyperthyroidism?

A201.- Graves

disease

Q202.- what is a common cause of hyperthyroidism in elderly pts?

A202.- toxic multinodular goiter- individual lumps, not a diffusely enlarged gla
nd

Q203.- viral infx with tender, painful thyroid gland is called...?

A203.- subacute thyroiditis

Q204.- classic lab pattern of hyperthyroidism?

A204.- low TSH (unless TSH-secreting tumor)- T3 and T4 are increased

Q205.- how do you treat hyperthyroidism?

A205.- propylthiouracil (PTU) and methimazole/carbimazole used as suppressive ag


ents

Q206.- what drug is used to treat a patient in


thyroid state)?

thyroid storm (ie. severe hyper

A206.- beta-blockers

Q207.- what is typical curative tx for hyperthyroidism? in pregnant pts?

A207.- radioactive iodine ablation of the gland- surgery in pregnant pts

Q208.- what is typical side effect of hyperthyroidism treatments? what is done?

A208.- hypothyroidism- thyroid hormone replacement

Q209.- weakness, lethargy, fatigue, cold intolerance, constipation, hair loss, h


oarseness, menorrhagia, memory impairment, dementia. these are sx of what endocr
ine abnormality?

A209.- hypothyroidism

Q210.- bradycardia; dry, coarse, cold, pale skin; periorbital and peripheral ede
ma; thick tongue; slow speech; slow reflexes. signs of what endocrine abnormalit
y?

A210.- hypothyroidism

Q211.- what is the most common cause of hypothyroidism? most common type of pati
ent?

A211.- Hashimoto s thyroiditis- women of reproductive age outnumber men 8:1

Q212.- non-tender goiter, lymphocytes in thyroid gland, antithyroid and antimicr


osomal antibodies. hypothyroid. disease?

A212.- Hashimoto s thyroiditis

Q213.- lab findings in hypothyroidism?

A213.- high TSH (unless due to 2* issues); low T3, low T4; hypercholesterolemia

Q214.- what lab findings are particular to Hashimoto s thyroiditis?

A214.- antithyroid and antimicrosomal antibodies

Q215.- why is it better to measure free T4 than total T4?

A215.- free T4 measures the active form of thyroid hormone; - many conditions ca
use changes in the amount of thyroid-binding globulin, which change total values
in the absence of hypo- or hyperthyroidism

Q216.- how is hypothyroidism treated?

A216.- T4- NOT T3

Q217.- what is euthyroid sick syndrome?

A217.- a patient with any illness may present with derangements in TFT s that re
semble hypothyroidism

Q218.- what are typical sx of patients with Cushing s syndrome? (increased corti
costeroids)

A218.- weight gain, - easy bruising, - acne, - hirsutism, - emotional lability,


- depression, - psychosis, - weakness, - memory loss

Q219.- what are typical signs of patients with Cushing s (incr. corticosteroids)
?

A219.- buffalo hump, truncal and central obesity with wasting of extremities, round plethoric faces, - purplish skin striae, - proximal muscle weakness

Q220.- what causes Cushing s syndrome (top 2 causes)?

A220.- 1. iatrogenic causes due to ubiquitous prescription of steroids- 2. Cushi


ng s disease (pituitary adenoma that secretes adrenocorticotropic hormone)

Q221.- is Cushing s disease more common in men or women?

A221.- Women of reproductive age outnumber men 5:1

Q222.- how do you diagnose Cushing s syndrome?

A222.- 1. 24hr measurement of free cortisol in urine (look for abnormal elevatio
n) or- 2. dexamethasone suppression test (cortisol levels aren t appropriately s
uppressed several hours after administration of dexamethasone)

Q223.- what is hirsutism?

A223.- male hair growth pattern in women or pre-pubescent children

Q224.- what is most common cause of hirsutism? what causes are important for boa
rds?

A224.- familial, genetic, or idiopathic hirsutism- Step 2: polycystic ovary synd


rome, Cushing s syndrome, drugs

Q225.- what 3 drugs cause hirsutism?

A225.- minoxidil- phenytoin- cyclosporine

Q226.- what is another name for polycystic ovary syndrome?

A226.- Stein-Levinthal syndrome

Q227.- what do you see in virilization?

A227.- clitoral enlargement- deepening of the voice- temporal balding

Q228.- what do you think about in a patient presenting with both hirsutism AND v
irilization?

A228.- androgen-secreting ovarian tumor: Sertoli-Leydig cell tumor or arrhenobla


stoma

Q229.- woman presents with diffuse goiter and hyperthyroidism. What are the expe
cted values of TSH and thyroid hormones?

A229.- low TSH & high thyroid hormones

Q230.- 48y/o female presents with progressive lethargy and extreme sensitivity t
o cold temperatures. What is the diagnosis

A230.- hypothyroidism

Q231.- pt with elevated serum cortisol levels undergoes a dexamethasone does not
decrease cortisol levels, but 8 mg does. What is the disease?

A231.- pituitary tumor

Q232.- 50y/o man complains of diarrhea. On physical exam his face is plethoric a
nd a heart murmur is detected. What is the dx?

A232.- carcinoid syndrome

Q233.- woman of short stature presents with shortened 4th and 5th metacarpals. W
hat endocrine d/o comes to mind?

A233.- Albright hereditary osteodystrophy or pseudohypoparathyroidism

Q234.- Nondiabetic patient presents with hypoglycemia but low levels of C peptid
e. What is the dx?

A234.- Surreptitious insulin injection

Q235.- cortex of the adrenal glands comes from what embryologic source?

A235.- mesoderm

Q236.- medulla of the adrenal glands comes from what embryologic source?

A236.- neural crest

Q237.- GFR corresponds to

A237.- Salt (Na+)- Sugar (glucocorticoids) and- Sex (androgens)- mneu: the deepr
you go the sweeter it gets

Q238.- What does GFR stand for?

A238.- zona glomerulosa- zona fasciculata- zona reticularis

Q239.- what does the zona glomerulosa secrete

A239.- aldosterone

Q240.- what is the primary regulatory control of the zona glomerulosa

A240.- renin-angiotensin

Q241.- what does the zona fasciculata secrete

A241.- cortisol, sex hormone

Q242.- what is the primary regulatory control of the zona fasciculata

A242.- ACTH, hypothalamic CRH

Q243.- what does the zona reticularis secrete

A243.- sex hormones (e.g., androgens)

Q244.- what is the primary regulatory control of the zona reticularis

A244.- ACTH, hypothalamic CRH

Q245.- what do the chromaffin cells of the medulla secrete

A245.- catecholamines (epi, NE)

Q246.- what is the primary regulatory control of the medulla

A246.- preganglionic sympathetic fibers

Q247.- what is the most common tumor of the adrenal medulla in adults

A247.- pheochromocytoma

Q248.- what is the most common tumor of the adrenal medulla in children

A248.- neuroblastoma

Q249.- out of pheochromocytoma and neuroblastoma which one causes episodic hyper
tension

A249.- pheochromocytoma

Q250.- how does the left adrenal gland drain blood

A250.- left adrenal ->left adrenal vein -> left renal vv -> IVC

Q251.- how does the right adrenal gland drain blood

A251.- right adrenal ->right adrenal vein -> IVC

Q252.- posterior pituitary is derived from what embryological origin

A252.- neuroectoderm

Q253.- posterior pituitary secretes what

A253.- vasopressen and oxytoxin

Q254.- where are vasopressen and oxytoxin made

A254.- hypothalamus

Q255.- where are vasopressen and oxytoxin shipped

A255.- pituitary

Q256.- where is the anterior pituatary derived from

A256.- oral ectoderm

Q257.- what does the anterior pituitary secrete

A257.- FLAT Pig- FSH- LSH- ACTH, Melanotropin (MSH)- TSH- Prolactin- GH

Q258.- alpha subunit is a common subunit to

A258.- TSH, LH, FSH, & hCG

Q259.- Beta subunit determines this

A259.- hormone specificity

Q260.- islet of Langerhans are collections of these three types of cells.

A260.- alpha, beta, delta cells

Q261.- what part of the pancreas are the alpha, beta, & delta cells most numerou
s

A261.- tail

Q262.- from where do the islets of Langerhans arise

A262.- pancreatic buds

Q263.- alpha cells secrete

A263.- glucagon

Q264.- beta cells secrete

A264.- insulin

Q265.- delta cells secrete

A265.- somatostatin

Q266.- prolactin ____ dopamine synthesis and secretion from the hypothalamus.

A266.- increases

Q267.- dopamine subsequently _______ prolactin secretion

A267.- decreases

Q268.- dopamine agonists (e.g., bromocriptine) ______ prolactin secretion. Dopam


ine antagonist (e.g., most antipsychotics) __________prolactin secretions.

A268.- decreases- increases

Q269.- in females prolactin inhibits_______ synthesis and release, which inhibit


s ovulation

A269.- GnRH

Q270.- ___________ this is commonly seen in prolactinomas

A270.- amenorrhea

Q271.- what is the congenital adrenal hyperplasia: Female that is not maturing p
resents with HYPERTENSION, hypokalemia, she has decreased sex hormones, decrease
cortisol, increased mineralocorticoids

A271.- 17 alpha hydroxylase deficiency

Q272.- this congenital adrenal hyperplasia is the most common form. It results i
n decreased cortisol (increased ACTH), decreased mineralocorticoids, increased s
ex hormones. Complications include masculinization, female psuedohermaphroditism
, Hypotension, hyponatremia, hyperkalemia, increased plasma renin activity, and
volume depletion. Salt wasting can need to hypovolemic shock in the newborn

A272.- 21 beta hydroxylase deficiency

Q273.- this congenital adrenal hyperplasia results in low cortisol, low aldoster
one and corticosterone and high sex hormones. Complications include masculinizat
ion and (Hypertension (11-deoxycorticosterone acts as a weak mineralocorticoid)

A273.- 11 beta hydroxylase deficiency

Q274.- where does PTH come from (cells and tissue)

A274.- chief cells of parathyroid

Q275.- what are the 4 fxs of PTH

A275.- 1) increased bone resorption of Ca++ & phosphate- 2) increased kidney res
orption of Ca++ in the distal convoluted tubule- 3) decreased kidney resorption
of phosphate- 4) increased 1,25 (OH)2 vitamin D (cholecalciferol) production by
stimulating kidney 1 alpha hydroxilase- mneu: PTH increases serum CA, decreases
serum PO, increases urine PO4, and stimulates both osteoclasts and osteoblastsPTH: phosphate trashing hormone

Q276.- what causes PTH secretion

A276.- decreased serum Ca++

Q277.- image p. 250 actions of PTH and 1,25 OH D in maitenence of Ca++ & phospha
te homeostasis

A277.- shown are the main actions of PTH & 1,25 (OH)2 D in the maintenance of Ca
++ and phosphate homeostasis

Q278.- source of vit D3

A278.- sun exposure in skin

Q279.- source of D2

A279.- plants

Q280.- where are D2 & D3 converted to 25 OH vit D

A280.- liver

Q281.- where is 25 OH vit D converted to 1,25 (OH)2 vit D (active form)

A281.- kidney

Q282.- what is the fx of vit D

A282.- 1)increased absorption of dietary Ca++- 2) increased absorption of dietar


y phosphate- 3)increased bone resorption of Ca++ & (PO4)3-

Q283.- what disease do you get if you don t get enough vit D (kids and adults)

A283.- kids-rickets- adults-osteomalacia

Q284.- what is 24,25-(OH)2 vit D

A284.- inactive form of vit D

Q285.- in the regulation of vit D, what does increased PTH cause

A285.- increase 1,25-(OH)2 vit D formation

Q286.- in the regulation of vit D, what does decreased [Ca++]cause

A286.- increase 1,25-(OH)2 vit D production

Q287.- in the regulation of vit D, what does increased 1,25-(OH)2 vit D cause

A287.- increase 1,25-(OH)2 vit D inhibits its own production

Q288.- What disease?- Ca++: - Phosphate:- Alk Phos:

A288.- hyperparathyroidism

Q289.- What disease?- Ca++: N/- Phosphate:N- Alk Phos:

A289.- paget s disease of bone

Q290.- What disease?- Ca++: - Phosphate:- Alk Phos:N/

A290.- vit D intoxication

Q291.- What disease?- Ca++: N- Phosphate:N- Alk Phos:N

A291.- osteoporosis

Q292.- What disease?- Ca++: - Phosphate:- Alk Phos:N

A292.- renal insufficiency

Q293.- where does calcitonin come from

A293.- parafolicular cells (C cells) of thyroid

Q294.- what is the fx of calcitonin

A294.- bone resorption of calcium

Q295.- calcitonin opposes the action of what hormone

A295.- PTH

Q296.- what stimulates calcitonin secretion

A296.- increased serum Ca++

Q297.- the need for gene transcription and protein synthesis delays the onset of
action of these steroid/thyroid hormones

A297.- mneu: PET CAT- Progesterone- Esterogen- Testosternone- Cortisol- Aldoster


one- Thyroxine and T3

Q298.- these hormones are lipophilic and insoluble in plasma; therfore, they mus
t circulate bound to specific binding globulins, which increase solubility and a
llows for increased delivery of steroid to the target organ.

A298.- steroid

Q299.- increased levels of sex hormone-binding globulin (SHBG) leads to lower le


vels of free testosterone and results in this physical exam finding

A299.- gynecomastia

Q300.- decreased SHBG raises free testosterone leading to this finding

A300.- hirsutism

Q301.- these are iodine containing hormones that control the body s metabolic ra
te

A301.- thyroid hormones (t3/t4)

Q302.- source of thyroid hormones

A302.- follicles of thyroid. Most T3 formed in the blood

Q303.- fx of thyroid hormones

A303.- 1) bone growth (synergism w/ GH- 2)CNS maturation- 3) beta adrenergic eff
ects (increased CO,HR,SV, contractility- 4) increase BMR via increased Na+/K+ AT
Pase activity -> increased O2 consumption, RR, increased body temp- 5) increased
glycogenolysis, gluconeogenesis, lipolysis- mneu: T3 fx--4Bs- Brain maturationBone growth- Beta adrenergic effects- BMR incrase

Q304.- what globulin binds most T3/T4 in blood

A304.- Throxine-binding globulin (TBG)

Q305.- is free or bound thyroid hormone active

A305.- free

Q306.- in what disease would you see decreased TBG

A306.- hepatic failure

Q307.- in what condition would you see increase TBG

A307.- pregnancy

Q308.- what hromone stimulates TSH secretion from the pituitary

A308.- TRH (hypothalamus

Q309.- what cells does TSH stimulate to secrete thyroid hormone

A309.- follicular cells

Q310.- negative feedback by this hormone to the anterior pituitary decreases sen
sitivity to TRH.

A310.- T3

Q311.- in Graves disease it is this TSH analog that stimulates follicular cells

A311.- TSI

Q312.- image thyroid hormone p. 252--

A312.- --

Q313.- what organs depend on insulin for glucose uptake

A313.- skeletal mm and adipose tissue

Q314.- what organs take up glucose independent of insulin levels (GLUT-1)

A314.- brain and RBC

Q315.- what fuel can the brain use in starvation states

A315.- ketone bodies

Q316.- this syndrome refers to increased cortisol levels due to a variety of cau
ses. The clinical picture includes hypertension, weight gain, moon faces, trunca
l obesity, buffalo hump, hyperglycemia (insulin resistance), skin changes (thinn
ing, striae), osteoporosis, amenorrhea, and immune suppression

A316.- Cushing syndrome

Q317.- If your patient presents with Cushings syndrome you have to identify the s
ource, if the patient has increased ACTH levels because of a pituitary adenoma t
he patient has

A317.- Cushing s disease

Q318.- If your patient presents with Cushings syndrome you have to identify the s
ource, if the patient has decreased ACTH levels due to causes that are not iatro
genic, the patient has

A318.- primary adrenal hyperplasia/neoplasia

Q319.- If your patient presents with Cushings syndrome you have to identify the s
ource, if the patient has decreased ACTH levels due to iatrogenic causes, the pa
tient may be taking

A319.- steroids (chronic)

Q320.- describe the dexamethasone suppression test in a healthy person

A320.- low cortisol after low dose

Q321.- describe the dexamethasone suppression test in a person with a ACTH-produ


cing tumor

A321.- high cortisol after low dose; low cortisol after high dose

Q322.- describe the dexamethasone suppression test in a person with a cortisone


producing tumor

A322.- high cortisol after low and high dose

Q323.- this disease is caused by an aldosterone-secreting tumor, resulting in HT


N, hypokalemia, metabolic alkalosis, and LOW plasma renin

A323.- primary hyperaldosteronism (Conn s syndrome)

Q324.- this disease is due to renal artery stenosis, chronic renal failure, cHF,
cirrhosis, or nephrotic syndrome. Kidney perception of low intravascular volume

results in an overactive renin0angiotensin system. Therefore, it is associated


with HIGH plasma renin

A324.- secondary hyperaldosteronism

Q325.- what is the tx for hyperaldosteronism

A325.- spironolactone (a K+ sparing diuretic that works by acting as an aldoster


one antagonist)

Q326.- this is a primary deficiency of aldosterone and cortisol due to adrenal a


trophy, causing hypotension (hyponatremic volume contraction) and skin hyperpigm
entation (due to MSH, a byproduct of increased ACTH production from POMC). It is
characterized by Adrenal Atrophy and Absence of hormone production. It involves
All 3 cortical divisions.

A326.- Addison s disease

Q327.- How you distinguish Addison s disease (primary aldosterone/cortisol defic


iency[renal problem]) from secondary aldosterone/cortisol deficiency [pituitary
problem]

A327.- Secondary insufficiency has no skin hyperpigmentation

Q328.- this is the most common tumor of the adrenal medulla in adults. It is der
ived from chromaffin cells which arise from the neural crest

A328.- pheochromocytoma

Q329.- what can pheochromocytomas be associated with if they occur in patients w

ith MEN types II & III

A329.- neurofibromatosis

Q330.- this is the most common tumor of the adrenal medulla in children, but it
can occur anywhere along the sympathetic chain

A330.- neuroblastoma

Q331.- what do pheochromocytomas secrete

A331.- epi, - NE, - dopamine

Q332.- how do you dx a pheo

A332.- urinary VMA and plasma catecholamines are elevated

Q333.- what endocrine d/os are pheos assoc with

A333.- MEN types II & III

Q334.- how do you tx pheos

A334.- alpha antagonists, - especially phenoxybenzamine, - a nonselective, - IRR


EVERSABLE alpha blocker

Q335.- symptoms of pheos occur in "spells"--meaning they often relapse and remit
. What are the 5 "P"s of a pheo

A335.- Pressure (elevated BP)- Pain (headache)- Perspiration- Palpitations (tach


ycardia)- Pallor

Q336.- give the rule of 10s associated with pheos

A336.- 10% malignant- 10% bilateral- 10% extraadrena- 10% calcify- 10% kids- 10%
familial

Q337.- this MEN type involves the 1) PANCREAS (e.g., Zollinger-Ellison syndrome,
insulinomas, VIPomas) 2)PARATHYROID 3) PITUITARY TUMORS. It often presents with
kidney stones and stomach ulcers

A337.- MEN I (Wermer s syndrome)= 3 "P" organs - (Pancreas, Pituitary, and Parat
hyroidism)

Q338.- this MEN type involves 1) MEDULLARY CARCINOMA OF THE THYROID 2)PHEOCHROMO
CYTOMA 3) PARATHYROID TUMOR.

A338.- MEN type II - (Sipple s syndrome)

Q339.- this MEN type involves 1) MEDULLARY CARCINOMA OF THE THYROID 2)PHEOCHROMO
CYTOMA 3) ORAL AND INTESTINAL GANGLIONEUROMATOSIS (MUCOSAL NEUROMAS).

A339.- MEN type III - ( formerly MEN type IIb )

Q340.- all MEN have this type of inheritance

A340.- autosomal dominant

Q341.- what gene are MEN type II & III associated with

A341.- ret gene

Q342.- this d/o often manifests itself with cold intolerance, hypoactivity, weig
ht gain, fatigue, lethargy, decreased appetite, constipation, weakness, decrease
d reflexes, facial/periorbital myxedema, dry, cool skin, and coarse, brittle hai
r

A342.- hypothyroidism

Q343.- what will the labs show in hypothyroidism

A343.- increased TSH (most sensitive test for primary hypothyroidism)- decreased
total T4- decreased free T4- decreased T3 uptake

Q344.- in this disease the patient presents hypothyroid. It results because thyr
oid is replaced by fibrous tissue

A344.- Riedel s thyroiditis

Q345.- this d/o manifests itself with heat intolerance, hyperactivity, weight lo
ss, chest pain/palpitations, arrhythmias, diarrhea, increased reflexes, warm moi
st skin, and fine hair

A345.- hyperthyroidism

Q346.- this is an autoimmune hyperthyroidism with thyroid stimulating/TSH recept


or antibodies. It often presents during stress (e.g., childbirth) with opthalmop
athy (proptosis, EOM swelling), pretibial myxedema, diffuse goiter.

A346.- graves disease

Q347.- what type of hypersensitivity reaction in graves disease

A347.- type II

Q348.- what lab results will you see in hyperthyroidism

A348.- decreased TSH (if primary)- increased total T4- increased free T4- increa
sed T3 uptake

Q349.- this is an autoimmune d/o resulting in hypothyroidism. It has a slow cour


se and often you will see a moderately enlarged, nontender thyroid. Microscopica
lly you will see a lymphocytic infiltrate with germinal genters. patient will te
st positive for antimicrosomal and antithyroglobulin antibodies

A349.- Hashimoto s thyroiditis

Q350.- this is a self-limited hypothyroidism which often follows a flulike illne


ss. patient will presents with jaw pain and a very tender thyroid gland. Labs wi
ll show signs of early inflammation and elevated ESR.

A350.- Subacute thyroiditis (de Quervain s)

Q351.- this is the most common thyroid cancer. It has an excellent prognosis. Mi
croscopically you will see "ground glass" nuclei and psammoma bodies

A351.- papillary carcinoma

Q352.- this thyroid cancer has a good prognosis. microscopically you will see un
iform follicules

A352.- follicular carcinoma

Q353.- this thyroid cancer arises from parafollicular "C cells". It produces cal
citonin. Microscopicallly you may see sheets of cells in amyloid stroma. It is s
een in MEN types II & III

A353.- medullary carcinoma

Q354.- this thyroid cancer occurs in older pts. It carries a very poor prognosis

A354.- undifferentiated/anaplastic

Q355.- this occurs wherever endemic goiter is prevalent (lack of dietary iodine)
. It is common in china. It can also be caused by a defect in T4 formation or de
velopmental failure in thyroid formation. You may see a pot-bellied, pale, puffy
-faced child with protruding umbilicus and protuberant tongue

A355.- cretinism

Q356.- adult patient presents with large tongue, deep furrows, deep voice, large
hands and feet, coarse facial features, impaired glucose tolerance (insulin res
istance). What is the d/o and what hormone is in excess

A356.- acromegly- increased growth hormone

Q357.- what does increased GH in children cause

A357.- gigantism

Q358.- how do you tx increased GH either in acromegaly or gigantism

A358.- octreotide

Q359.- give some natural states in which you might have increased GH

A359.- stress, - exercise, - and hypoglycemia

Q360.- this is usually caused by an adenoma. labs will show hypercalemia, hyperc
alciuria (RENAL STONES), hypophosphatemia, increased, PTH & cAMP in urine. While
it is often asymptomatic it may present with weakness and constipation ("GROANS
")

A360.- primary hyperparathyroidism

Q361.- this d/o is due to decreased serum Ca++, most often due to chronic renal
disease. Labs will show hypocalcemia, hyperphosphatemia, and increased PTH

A361.- secondary hyperparathyroidism (hyperplasia)

Q362.- this complication of primary hyperparathyroidism results when cystic bone


spaces are filled with brown fibrous tissue causing BONE PAIN

A362.- Osteitis fibrosa cystica (von recklinghausen s syndrome)

Q363.- this complication of secondary hyperparathyroidism, results in bone lesio


ns due to renal disease

A363.- renal osteodystrophy

Q364.- give the rhyme that describes hyperparathyroidism

A364.- stones, bones, and groans

Q365.- pt presents with hypocalcemia or tetany. what disease

A365.- hypoparathyroidism.

Q366.- give 2 causes of hypoparathyroidism

A366.- accidental surgical excision (thyroid sx)- DiGeorge syndrome

Q367.- Chvostek s sign is a sign of hypoparathyroidism. What is it?

A367.- tap facial nerve results in contraction of facial mm

Q368.- trousseau s sign is a sign of hypoparathyroidism. What is it?

A368.- occlusion of brachial artery w/ BP cuff results in carpal spasm

Q369.- pt presents w/ hypocalcemia, shortened stature and shortened 4th & 5th di
gits.

A369.- pseudohypoparathyroidism

Q370.- what is pseudohypoparathyroidism

A370.- kidneys are unresponsive to PTH

Q371.- what is the inheritance of pseudohypoparathyroidism

A371.- autosomal-recessive

Q372.- give 5 causes of hypercalcemia

A372.- mneu: CHIMPANZEES- Calcium ingestion, - Hyperparathyroid, - Hyperthyroid,


, - Iatrogenic (thiazides, - Multiple myeloma, - Paget s disease, - Addison s di
sease, - Neoplasms, - Zollinger-Ellison syndrome, - Excess vit D, - Excess vit A
, - Sarcoidosis

Q373.- give some acute manifestations of diabetes mellitus

A373.- polydipsia, - polyuria, - polyphagia, - weight loss, - DKA (type 1), - hy


perosmolar coma (type 2), - unopposed secretion of GH and epinephrine (exacerbat
ing hyperglycemia)

Q374.- Give some chronic manifestations of Diabetes Mellitus

A374.- small vessel disease (diffuse thickening of basement membrane) ->retinopa


thy (hemorrhage, exudates, microaneurysms, vessel proliferation), claucoma, neph
ropathy (nodular sclerosis, progressive protinuria, chronic renal failure, arter
iosclerosis leading to HTN)- 2. Large vessel atherosclerosis, CAD, peripheral va
scular occlusive disease and gangrene, cerebrovascular disease- Neuropathy (moto
r, sensory, and autonomic degeneration)- Cataracts (sorbitol accumulation

Q375.- what tests should you run in dbts

A375.- fasting serum glucose- glucose tolerance test- HbA1c

Q376.- what does HbA1c measure

A376.- long term diabetic control

Q377.- what is the primary defect in type 1--juvenile onset (IDDM)

A377.- viral or immune destruction of beta cells

Q378.- what is the primary defect in type 2--adult onset (NIDDM)

A378.- increased resistance to insulin

Q379.- is insulin necessary for tx in type 1--juvenile onset (IDDM)

A379.- always

Q380.- is insulin necessary for tx in type 2--adult onset (NIDDM)

A380.- sometimes

Q381.- at what age does type 1--juvenile onset (IDDM)usually strike [many except
ions]

A381.- <30

Q382.- at what age does type 2--adult onset (NIDDM)usually strike [many exceptio
ns]

A382.- >40

Q383.- is type 1--juvenile onset (IDDM)associated with obesity

A383.- no

Q384.- is type 2--adult onset (NIDDM)associated with obesity

A384.- yes

Q385.- is there a genetic predisposition to type 1--juvenile onset (IDDM)

A385.- weak, polygenic

Q386.- is there a genetic predisposition to type 2--adult onset (NIDDM)

A386.- strong, polygenic

Q387.- is there an HLA association to type 1--juvenile onset (IDDM)

A387.- yes (HLA-DR3 &4)

Q388.- is there an HLA association to type 2--adult onset (IDDM)

A388.- no

Q389.- describe the glucose intolerance associated with type 1--juvenile onset (
IDDM)

A389.- severe

Q390.- describe the glucose intolerance associated with type 2--adult onset (NID
DM)

A390.- mild-moderate

Q391.- Is ketoacidosis associated with type 1--juvenile onset (IDDM)

A391.- commonly

Q392.- Is ketoacidosis associated with type 2--adult onset (NIDDM)

A392.- rarely

Q393.- describe the beta cell numbers in the islets associated with type 1--juve
nile onset (IDDM)

A393.- low

Q394.- describe the beta cell numbers in the islets associated with type 2--adul
t onset (NIDDM)

A394.- variable

Q395.- describe the serum insulin associated with type 1--juvenile onset (IDDM)

A395.- low

Q396.- describe the serum insulin associated with type 2--adult onset (NIDDM)

A396.- variable

Q397.- are the classic symptoms of polyuria, polydipsia, thirst, and weight loss
associated with type 1--juvenile onset (IDDM)

A397.- commonly

Q398.- are the classic symptoms of polyuria, polydipsia, thirst, and weight loss
associated with type 2--adult onset (NIDDM)

A398.- sometimes

Q399.- diabetic ketoacidosis is one of the most important complications of type


1 diabetes describe its pathology

A399.- excess fat breakdown and increased ketogenesis from increase in free fatt
y acids, which are then made into ketone bodies.

Q400.- what is ketoacidosis usually due to

A400.- an increase in insulin requirements due to an increase in stress (e.g., i


nfection)

Q401.- give some signs and symptoms of diabetic ketoacidosis

A401.- Kussmaul respirations (rapid/deep breathing), - hyperthermia, - nausea/vo


miting, - abdominal pain, - psychosis/dementia, - dehydration. - Fruity breath o
dor (due to exhaled acetone)

Q402.- what will the labs of a patient in diabetic ketoacidosis show

A402.- hyperglycemia, - increase H+, - decreased HCO3-, (anion gap metabolic aci
dosis, - increased blood ketone levels, - leukocytosis. - Hyperkalemia, but depl
eted intracellular K+

Q403.- what are some complications of diabetic ketoacidosis

A403.- life-threatening mucormycosis, - rhizopus infection, - cerebral edema, cardiac arrhythmias, - heart failure

Q404.- how do you tx a patient in diabetic ketoacidosis

A404.- fluids, insulin, and potassium; glucose if necessary to prevent hypoglyce


mia

Q405.- this d/o is characterized by intensive thirst and polyuria together with
an inability to concentrate urine owing to lack of ADH

A405.- diabetes insipidus

Q406.- this type of diabetes insipidus may be caused by a pituitary tumor, traum

a, surgery, histiocytosis

A406.- central DI

Q407.- this type of diabetes insipidus may be caused by hereditary factors, hype
rcalcemia, lithium, demeclocycline

A407.- nephrogenic DI (lack of renal response to ADH)

Q408.- how do you dx DI

A408.- water deprivation test --urine specific gravity <1.006- serum osmo >290mO
sm/L

Q409.- how do you tx central DI

A409.- intranasal despopressin (ADH analog

Q410.- this d/o results in- 1)excessive water retention- 2)hyponatremia- 3)urine
osmolarity>serum osmolarity

A410.- syndrome of inappropriate antidiuretic hormone secretion

Q411.- how do you tx SIADH

A411.- demeclocycline or H2O restriction- note: tx slowly

Q412.- in SIADH very low serum sodium levels can lead to this

A412.- seizures

Q413.- give some causes of SIADH

A413.- 1) ectopic ADH (small cell lung CA)- 2) CNS disorders/head trauma- 3) pul
monary disease- 4) Drugs (e.g., cyclophosphamide)

Q414.- this is a rare syndrome that results in recurrent diarrhea, cutaneous flu
shing, asthmatic wheezing, and right sided valvular disease.

A414.- carcinoid syndrome

Q415.- what will you find in the urine of a patient with carcinoid syndrome

A415.- increased 5-HIAA in urine

Q416.- carcinoid tumor is the most common tumor in this part of large intestine

A416.- appendix

Q417.- carcinoid tumors result from this type of cell

A417.- neuroendocrine cells

Q418.- carcinoid tumor secretes high levels of this

A418.- 5-HT (seratonin)

Q419.- if carcinoid tumor is limited to here you wont see symptoms

A419.- GI tract- 5-HT undergoes 1st pass metabolism in liver

Q420.- give the rule of 1/3 for carcinoid tumors

A420.- 1/3 metastasize- 1/3 present w/ 2nd malignancy- 1/3 multiple

Q421.- where to carcinoid tumors derive from

A421.- neuroendocrine cells of GI tract

Q422.- how do you tx carcinoid tumors

A422.- octreotide

Q423.- this is a gastrin secreting tumor of pancreas or duodenum. It causes recu


rrent ulcrs

A423.- Zollinger-Ellison syndrome

Q424.- what MEN syndrome is ZE syndorme associated with

A424.- MEN type I

Q425.- what is the tx strategy for type 1 DM

A425.- low sugar diet, insulin replacement

Q426.- what is the tx strategy for type 2 DM

A426.- dietary modification, exercise for weight loss, and oral hypoglycemics

Q427.- What are the functions of cortisol?

A427.- Maintains blood pressure (up regulates alpha 1 receptors on arterioles),


- decreases Bone formation, - anti-Inflammatory, - decreases Immune function, increases Gluconeogenesis (also lipolysis and proteolysis)

Q428.- What is the #1 cause of Cushing s syndrome?

A428.- Exogenous steroids

Q429.- In what cases of Cushing s syndrome would you expect to see increased ACT
H? decreased ACTH?

A429.- Increased ACTH - pituitary adenoma, nonpituitary tissue making ACTH (smal
l cell lung cancer)- Decreased ACTH - exogenous steroids, adrenal adenoma produc
ing cortisol; these are due to negative feedback

Q430.- What are the symptoms of Cushing s syndrome?

A430.- Hypertension, - weight gain, - moon faces, - truncal obesity, - buffalo h


ump, - hyperglycemia (insulin resistance), - skin changes (thining, striae), - o
steoporosis, - amenorrhea, - and immune suppression

Q431.- What drug is given in order to determine the location of production of co


rtisol in Cushing s syndrome?

A431.- Dexamethasone

Q432.- What is the difference between Addison s disease and secondary adrenal in
sufficiency?

A432.- Addison s presents with skin hyperpigmentation and hyperkalemia; - second


ary adrenal insufficiency does not.- Hypotension and hyponatremia can still be p
resent in both.

Q433.- What are functions of aldosterone?

A433.- Na+ reabsorption, K+ secretion (indirectly), H+ secretion

Q434.- Conn s syndrome

A434.- Primary hyperaldosteronism- Caused by aldosterone secreting tumor, result


ing in hypertension, hypokalemia, metabolic alkalosis, and LOW plasma renin. May
be bilateral or unilateral.- Treatment includes spironolactone, a K+ sparing di
uretic that works by acting as an aldosterone antagonist.

Q435.- Secondary hyperaldosteronism

A435.- Kidney perception of low intravascular volume results in an overactive re


nin-angiotensin system. - Due to renal artery stenosis, chronic renal failure, C
HF, cirrhosis, or nephrotic syndrome. - Associated with HIGH plasma renin.

Q436.- What is the most common cause of adrenal insufficiency?

A436.- Sudden withdrawal of corticosteroids

Q437.- 17 alpha hydroxylase deficiency

A437.- Decreased sex hormones, decreased cortosol, increased mineralocorticoids.


- Symptoms include hypertension and hypokalemia. - XY - decreased DHT causes ps
eudohermaphroditism (externally phenotypic female, no internal reproductive stru
ctures due to MIF). - XX - externally phenotypic female with normal sex organs,
but lacking secondary sexual characteristics ("sexual infantilism")

Q438.- 21 hydroxylase deficiency

A438.- Decreased cortisol and mineralocorticoid production, increased sex hormon


e production. - Symptoms include masculinization, female pseudohermaphroditism,
hypotension, hyperkalemia, increased plasma renin activity, and volume depletion
. Salt wasting can lead to hypovolumetric shock in the newborn.

Q439.- 11 beta hydroxylase deficiency

A439.- Decreased cortisol, - decreased aldosterone and corticosterone, - increas


ed sex hormones.- Symptoms include masculinization, hypertension (11-deoxycortic
osterone is a mineralcorticoid and is secreted in excess).

Q440.- The alpha subunit is common in what anterior pituitary secretions?

A440.- FSH, - LH, - TSH, - and GH

Q441.- Which endocrine hormones have a tyrosine kinase signaling pathway?

A441.- Insulin, IGF-1, FGF, PDGF, Prolactin, and GH- Think all the growth stuff.
.. and prolactin.

Q442.- How is insulin secreted?

A442.- Glucose enters beta cells of the pancreas through GLUT-2 receptor. - ATP
from glucose metabolism closes K+ channels and depolarizes cell. - Ca2+ channels
open, and insulin is released.

Q443.- Diazoxide

A443.- Keeps Ca2+ channels open and inhibits insulin release. - Used to treat in
sulinomas.

Q444.- What affect does TRH (Thyotropin releasing hormone) have on the anterior
pituitary?

A444.- Increases TSH and prolactin release

Q445.- What affect does somatostatin have on the anterior pituitary?

A445.- Decreases GH and TSH release

Q446.- What is used in the treatment of a prolactinoma?

A446.- Dopamine agonist, like bromocriptine; - dopamine antagonists, like antips


ychotics, stimulate prolactin secretion.

Q447.- Calcitonin

A447.- From the parafollicular cells (C cells) of thyroid. - Secreted in respons


e to increased serum calcium, and causes decreased bone resorption.

Q448.- 3 beta hydroxysteroid dehydrogenase deficiency

A448.- Inability to produce glucocorticoids, mineralocorticoids, androgens, or e


strogens. - Causes an early death and excessive salt excretion.

Q449.- What effect does cortisol have on blood pressure?

A449.- Cortisol maintains blood pressure by upregulating alpha 1 receptors on ar


terioles.

Q450.- What are common causes of decreased magnesium secretion? What effect does
low serum magnesium have on PTH?

A450.- Common causes of decreased magnesium include diarrhea, aminoglycosides, d


iuretics, and alcohol abuse. - Low serum magnesium inhibits secretion of parathy
roid hormone.

Q451.- What affect does PTH have on calcitriol levels?

A451.- PTH increases calcitriol levels by stimulating kidney 1 alpha hydroxylase


.

Q452.- What are our sources for vitamin D3 and D2?

A452.- D3 - sun exposure in skin; D2 - ingested from plants

Q453.- Where is calcitonin produced?

A453.- Parafollicular cells of thyroid

Q454.- Which endocrine hormones use the cGMP signaling pathway?

A454.- ANP and NO (EDRF)- Think vasodilators

Q455.- Which endocrine hormones use the IP3 signaling pathway?

A455.- GnRH, GHRH, Oxytocin, ADH, TRH- Anything made in the hypothalamus, not th

e pituitary.

Q456.- Which endocrine hormones use a tyrosine kinase signaling pathway?

A456.- IGF-1, FGF, PDGF, Insulin - MAP kinase- Prolactin and GH - JAK/STAT- cyto
kines and IL-2 also use JAK/STAT pathway

Q457.- What effect does an increase in sex hormone binding globulin have in men?

A457.- Gynecomastia - due to lower levels of free testosterone; - only free test
osterone has an effect.

Q458.- What are the effects of T3?

A458.- Brain maturation, Bone growth, Beta-adrenergic effects, and increased Bas
al metabolic rate.- Think the 4 Bs!

Q459.- What effect does estrogen have on thyroid binding globulin?

A459.- Estrogen increases thyroid binding globulin; - TBG is therefore increased


in pregnancy and OCP use.- T4 is also increased, but free T4 is normal (due to
increased TBG holding onto the increased amount).

Q460.- What enzyme is responsible for oxidation and organification of iodide as


well as coupling of MIT and DIT?

A460.- Peroxidase

Q461.- How can Conn s syndrome be distinguished from secondary hyperaldosteronis


m?

A461.- Conn s syndrome with have low renin, - while secondary hyperaldosteronism
will have high renin (due to lack of blood flow to kidney).

Q462.- What will be found elevated in the urine in a case of pheochromocytoma?

A462.- VMA

Q463.- What conditions is pheochromocytoma associated with?

A463.- Neurofibromatosis, MEN 2A, MEN 2B, diabetes mellitus- Can also cause poly
cythemia due to excess erythropoiten being produced.

Q464.- Neuroblastoma

A464.- Most common tumor of the adrenal medulla in children. - Can occur anywher
e along the sympathetic chain. HVA is urine. - Less likely to develop hypertensi
on.- Accociated with N-myc oncogene. - Tumor marker is bombasin. - Stain is neur
ofilament stain. - Histology would show Homer-wright pseudorosettes.

Q465.- What condition is associated with HLA-DR5 and Hurthle cells?

A465.- Hashimoto s thyroiditis

Q466.- What are the findings in cretinism?

A466.- Pot-bellied, pale, puffy faced child with protruding umbilicus and protub
erant tongue.

Q467.- What often precedes subacute (de Quervain s) thyroiditis?

A467.- Flu-like illness- Associated with elevated ESR, jaw pain, early inflammat
ion, and VERY TENDER thyroid gland. - Histology shows granulomatous inflammation
.

Q468.- Jod-Baselow phenomenon

A468.- Thyrotoxicosis if a patient with iodine deficiency goiter is made iodine


replete.

Q469.- Papillary carcinoma of the thyroid

A469.- Most common, - excellent prognosis, - "ground glass" nuclei (Orphan Annie
), - psammoma bodies, - nuclear grooves. - Increased risk with childhood irradat
ion.

Q470.- Medullary carcinoma of thyroid

A470.- From parafollicular "C cells"; - produces calcitonin, sheets of cells in


amyloid stroma. - Associated with MEN types 2A and 2B. - Usually sporadic.

Q471.- Albright s hereditary osteodystrophy

A471.- Autosomal dominant kidney unresponsiveness to PTH. - Hypocalcemia, shorte

ned 4th/5th digits, short stature.

Q472.- What drug when given in a case of diabetes insipidus, can distinguish bet
ween cental and nephrogenic?

A472.- Desmopression; - central DI will cause concentrated urine when desmopress


ion is administered, while nephrogenic will not.

Q473.- What is the drug of choice for diabetes insipidus?

A473.- Hydrochlorothiazide- Can also used indomethacin or amiloride (if due to l


ithium).

Q474.- What is the treatment for SIADH

A474.- Demeclocycline (tetracycline antibiotic) or H2O restriction

Q475.- Which HLA types is diabetes mellitus type 2 associated with?

A475.- HLA-DR3 and HLA-DR4

Q476.- What are some of the complications of diabetic ketoacidosis?

A476.- Life-threatening mucormycosis, - Rhizopus infection, - cerebral edema, cardiac arrhythmias, - and heart failure

Q477.- What is the treatment of DKA?

A477.- Fluids, insulin, and K+ (to replenish stores); - glucose if necessary to


prevent hypoglycemia.

Q478.- What are some of the symptoms of carcinoid syndrome?

A478.- Diarrhea, - cutaneous flushing, - asthmatic wheezing, - and right sided v


alvular disease.

Q479.- What is the treatment for carcinoid syndrome?

A479.- Octreotide

Q480.- MEN 1

A480.- Parathyroid tumors, - Pituitary tumors (prolactin or GH), and - Pancreati


c endocrine tumors (Zollinger-Ellison syndrome, insulinoma, VIPomas, glugagonoma
s)- Commonly presents with kidney stones and stomach ulcers. - 3 Ps

Q481.- MEN 2A

A481.- Medullary thyroid cancer (secretes calcitonin), - Parathyroid cancer, - P


heochromocytoma- 2 Ps- Associated with ret gene

Q482.- MEN 2B

A482.- Pheochromocytoma, - Medullary thyroid carcinoma (secretes calcitonin), -

Oral/intestinal ganglioneuromatosis (associated with marfanoid habitus).- 1 P- A


ssociated with ret gene.

Q483.- Papillary thyroid carcinoma

A483.- Most common thyroid malignancy; - excellent prognosis, "ground glass" nuc
lei (Orphan Annie), psammoma bodies, nuclear grooves. - Increased risk with chil
dhood irradiation.

Q484.- Follicular thyroid carcinoma

A484.- Good prognosis, uniform follicles; - 40% have Ras mutation.

Q485.- Medullary thyroid carcinoma

A485.- From parafollicular "C cells"; - produces calcitonin, sheets of cells in


amyloid stroma. - Associated with MEN type 2A and 2B, however it s usually spora
dic.

Q486.- Low serum calcium and phosphate and elevated alkaline phosphatase suggest
the diagnosis of...

A486.- osteomalacia (low levels of vitamin D results in low calcium and phosphat
e, and the elevated alk phos is a reflection of increased osteoblast activity)

Q487.- Paget s disease effect on alkaline phosphatase

A487.- Elevated due to intense osteoblast activity

Q488.- Osteoporosis lab values of calcium, phosphate, and alkaline phosphatase

A488.- Normal calcium and phosphate levels, and alkaline phosphatase increases a
fter a fracture

Q489.- Hyperparathyroidism lab values of calcium and phosphate

A489.- Increased calcium, - decreased phosphate

Q490.- Hypoparathyroidism lab values of calcium and phosphate

A490.- Decreased calcium, - increased phosphate

Q491.- Thyroid replacement therapy effect on a patient who is on an oral hypogly


cemic

A491.- Serum levels of the oral hypoglycemic may increase, requiring a lower dos
e of the agent

Q492.- Primary hyperthyroidism effects on thyroid function blood tests

A492.- Decreased TSH, - increased T4 and T3

Q493.- The single best test for assessing thyroid function

A493.- TSH. High TSH = hypothyroidism. - Low TSH = hyperthyroidism.

Q494.- Primary hypothyroidism effects on thyroid function blood tests

A494.- Increased TSH, - decreased T3 and T4

Q495.- Most common cause of hypothyroidism

A495.- Hashimoto s thyroiditis

Q496.- Complication of severe hypothyroidism

A496.- Myxedema coma - (decreased mental status, hypothermia, and other parasymp
athetic symptoms. Mortality is 30-60%)

Q497.- Hyperthyroidism with exopthalmos, pretibial myxedema, and thyroid bruits

A497.- Graves

disease

Q498.- Mainstay of treatment for hyperthyroidism

A498.- Radioactive thyroid ablation. - If contraindicated, then thyroidectomy or


antithyroid drugs (methimazole or propylthiouracil)

Q499.- Best method of assessing a thyroid nodule for malignancy

A499.- Fine needle aspiration

Q500.- Type of thyroid carcinoma that is associated with MEN II

A500.- Medullary thyroid carcinoma

Q501.- MOA of repaglinide (of the meglitinide class of oral hypoglycemics)

A501.- Stimulates pancreatic beta cells to secrete insulin

Q502.- Treatment of hyperthyroidism in patients who are pregnant

A502.- Antithyroid drugs such as propylthiouracil (PTU) or methimazole (tapazol)

Q503.- Treatment for thyroid storm

A503.- Beta-blocker to control arrhythmia and tachycardia, antithyroid drugs (PT


U or methimazole), - verapamil for severe heart failure in the presence of asthm
a or arrhythmia

Q504.- Thyroid lab values in thyroid storm

A504.- Elevated T4, free T4, and T3, - and low TSH

Q505.- Symptoms of hyperthyroidism with hyperpyrexia, heart failure, shock, coma

, or death

A505.- Thyroid storm. - Commonly occurs after surgery, radioactive iodine therap
y, or during a severe illness such as uncontrolled diabetes, trauma, acute infec
tion, severe drug reaction, or MI

Q506.- Secondary hypothyroidism is due to...

A506.- Pituitary TSH deficit. - Characterized by normal or low TSH and low T4

Q507.- Elevated TSH, decreased free T4, and elevated thyroid peroxidase antibodi
es (TPO) and microsomal thyroid antibodies

A507.- Hashimoto s thyroiditis

Q508.- Cautions in initiating levothyroxine therapy

A508.- Use lower starting dose in cardiac disease as it may exacerbate angina. Concomitant administration of cholestyramine, antacids, and iron supplements ma
y interfere with absorption.

Q509.- Most aggressive form of thyroid cancer

A509.- Anaplastic carcinoma

Q510.- Name the 4 types of thyroid cancer

A510.- Papillary, - follicular, - medullary, - and anaplastic

Q511.- PTH can be suppressed by excess amounts of what vitamin?

A511.- Vitamin D (at a dosage >50,000 IU)

Q512.- Treatment for hyperparathyroidism

A512.- Calcitonin to suppress PTH secretion, reducing bone resorption. - More ag


gressive therapy includes pamidronate or bisphosphanates to inhibit osteoclast b
one resorption. - Plicamycin for refractory patients.

Q513.- Immediate treatment for very high calcium (>14)

A513.- IV hydration with normal saline, diuresis with IV furosemide, monitor and
replace potassium and magnesium.

Q514.- Sudden onset (1-3 weeks) of polyuria, polydypsia, weakness/fatigue, polyp


hagia, weight loss, and nocturnal diuresis.

A514.- Type I diabetes

Q515.- Type I diabetes basic principles of insulin therapy

A515.- Split doses of rapid acting (lispro or regular) and intermediate-acting (


NPH or Lente) insulin BID. - Adjust according to home blood sugar values.

Q516.- Lab criteria for the diagnosis of DM II

A516.- Fasting blood sugar >126 on 2 occasions, 2-hour postprandial glucose >200
after 75g oral glucose, or random glucose >200 on 2 occasions with sxs of diabe
tes.

Q517.- MOA of sulfonylureas such as glyburide

A517.- Stimulates pancreatic beta cells to secrete insulin and may increase numb
er of insulin receptors on hepatocytes, improving glucose uptake. - (Reduce bloo
d glucose by 70-80)

Q518.- Sulfonylurea major contraindication

A518.- Pregnancy.

Q519.- Major side effect of sulfonylureas

A519.- Hypoglycemia

Q520.- MOA of biguanides (metformin)

A520.- Decrease hepatic gluconeogenesis

Q521.- Side effects of metformin

A521.- GI upset, - lactic acidosis, - and decreased B12 absorption

Q522.- MOA of thiazoladinediones (TZDs)

A522.- Improve insulin sensitivity in muscle and fat cells and may inhibit hepat
ic glucose output

Q523.- MOA of alpha-glucosidase inhibitors

A523.- Decrease postprandial hyperglycemia by decreasing the rate of absorption


of most carbohydrates by binding more readily to intestinal disaccharides than d
igested carbohydrate products.

Q524.- Common side effect of alpha-glucosidase inhibitors (Acarbose)

A524.- Flatulence

Q525.- C-peptide levels in type 2 diabetes

A525.- Normal or elevated

Q526.- C-peptide levels in type 1 diabetes

A526.- Low

Q527.- Purpose of measuring c-peptide levels in patients who are injecting insul
in

A527.- Determine how much of their own insulin they are producing

Q528.- What do low levels of C-peptide and hypoglycemia indicate?

A528.- Patients who are abusing insulin, a sign of malingering.

Q529.- Natural physiologic functions of C-peptide

A529.- 1. Repairs the muscular layer of arteries- 2. Prevents diabetic nephropat


hy, and improves blood flow to the heart

Q530.- 2 fundamental pathogenic defects that are the hallmarks of type 2 diabete
s

A530.- Insulin resistance and impaired insulin secretion

Q531.- The liver s response to decreased insulin production by pancreatic beta c


ells.

A531.- Increased glucose production. - (The loss of the insulin inhibitory effec
t on the liver causes an increase in hepatic glucose output)

Q532.- The first step and cornerstone in the management of type 2 diabetes

A532.- Lifestyle interventions aimed at weight reduction and increased physical


activity in order to improve insulin sensitivity.

Q533.- Name the classes of oral antidiabetic agents that are considered insulin
secretagogues

A533.- Sulfonylureas, - meglitinides, - and dipeptidyl peptidase-IV inhibitors (


sitagliptin aka Januvia).

Q534.- MOA of incretin mimetics (exenatide aka Byetta)

A534.- Reduces hyperglycemia by stimulating glucagon-like peptide (GLP)-1 recept


ors only when levels of glycemia exceed basal levels. - Administered subcutaneou
sly.

Q535.- Low HDL, high triglycerides, high uric acid levels, central obesity, fast
ing hyperglycemia, and high blood pressure

A535.- Metabolic syndrome (syndrome X)

Q536.- ADA recommended target for hemoglobin A1C in diabetic patients

A536.- <7.0

Q537.- Test to detect postprandial hyperglycemia in patients with impaired gluco


se tolerance but no symptoms of diabetes

A537.- 2 hour oral glucose tolerance test

Q538.- Most common complication and cause of death in patients with type 2 diabe

tes

A538.- Cardiovascular disease

Q539.- ADA recommendations for HDL level in female patients with type 2 diabetes
.

A539.- >50

Q540.- Recommended annual assessments in patients with type 2 diabetes

A540.- Dilated eye exam, - urinary albumin measurement, - and comprehensive foot
exam

Q541.- Regular insulin: Onset of action, peak effect, and duration

A541.- Onset: 30-60 mins- Peak: 2-4 hours- Duration: 5-8 hours

Q542.- Humalog (Lispro): Onset, peak, and duration

A542.- Onset: 5-10 minutes- Peak: 0.5-1.5 hours- Duration: 6-8 hours

Q543.- Novalog (Aspart): Onset, peak, and duration

A543.- Onset: 10-20 minutes- Peak: 1-3 hours- Duration: 3-5 hours

Q544.- Apidra (glulisine): Onset, peak, and duration

A544.- Onset: 5-15 minutes- Peak: 1-1.5 hours- Duration: 1-2.5 hours

Q545.- NPH insulin: Onset, peak, and duration

A545.- Onset: 2-4 hours- Peak: 6-10 minutes- Duration: 18-28 hours

Q546.- Levemir (detemir): Onset, peak, and duration

A546.- Onset: 2 hours- Peak: Not discernible- Duration: 20 hours

Q547.- Lantus (glargine): Onset, peak, and duration

A547.- Onset: 1-4 hours- Peak: Not discernible- Duration: 20-24 hours

Q548.- Diagnostic study to evaluate for adrenal insufficiency

A548.- ACTH stimulation test: shows decreased morning serum cortisol and increas
ed ACTH

Q549.- Etiology of secondary adrenal insufficiency

A549.- Decreased ACTH production by the pituitary, caused by cessation of long-t


erm glucocorticoid treatment

Q550.- Autoimmune adrenal cortical destruction leading to defiencies of miraloco


rticoids and glucocorticoids

A550.- Addison s disease

Q551.- Increased ACTH secretion can cause what dermatologic manifestation?

A551.- Hyperpigmentation. - Seen in Addison s disease.

Q552.- 4 S s of adrenal crisis management?

A552.- Salt: 0.9% saline- Steroids: IV hydrocortisone 100mg q8hours- Support- Se


arch for underlying illness

Q553.- 2 types of corticosteroids

A553.- glucocorticoids (cortisol) and mineralocorticoids (aldosterone)

Q554.- What type of corticosteroid is dexamethasone?

A554.- Almost purely glucocorticoid

Q555.- What type of corticosteroid is prednisone?

A555.- Mixed glucocorticoid and mineralocorticoid action

Q556.- 5 P s of Pheochromocytoma

A556.- Pressure (BP)- Pain (headache)- Perspiration- Palpitations- Pallor/diapho


resis

Q557.- Unilateral adrenal adenoma

A557.- Conn s syndrome

Q558.- Hyperaldosteronism presentation

A558.- hypertension, headache, polyuria, and muscle weakness. - In severe cases,


may have tetany, parasthesias, and peripheral edema

Q559.- Treatment for hyperaldosteronism

A559.- Lap or open adrenalectomy for adrenal tumors after correcting BP and pota
ssium. - Treat with spironolactone for bilateral adrenal hyperplasia.

Q560.- Is hypothyroidism associated with hypo- or hypernatremia?

A560.- Hyponatremia. - Obtaining a TSH is part of the evaluation for hyponatremi


a.

Q561.- Thyroid disease that presents with decreased cognitive functioning (memor
y) and depression

A561.- Hypothyroidism

Q562.- What are the causes of primary hyperparathyroidism?

A562.- Multiple endocrine neoplasia, - excessive parathyroid hormone

Q563.- What are the causes of secondary hyperparathyroidism?

A563.- Renal failure, metastatic bone disease, osteomalacia, and multiple myelom
a

Q564.- What are the symptoms of hyperparathyroidism?

A564.- "moans, groans, stones, bones, and psychiatric overtones"- Moans: not fee
ling well- groans: abdominal pain, constipation- stones: kidney stones- bones: b
one pain and arthralgias- psychiatric overtones: lethargy, fatigue, depression,
memory problems

Q565.- Lab values show hypercalcemia and hypophosphatemia. What should be ruled
out?

A565.- Hyperparathyroidism

Q566.- What is the most common cause of hypoparathyroidism?

A566.- Thyroidectomy

Q567.- Labs reveal increased T3 and Free T4, decreased TSH, and elevated antimic
rosomal and antithyroglobulin antibody titers. Diagnosis?

A567.- Graves disease

Q568.- What medication is used to treat the symptoms of Graves disease until th
e hyperthyroidism is resolved?

A568.- Propanolol

Q569.- What 3 drugs can lead to hypothyroidism?

A569.- lithium, - sulfonamides, - and amiodarona

Q570.- What type of hypothyroidism is precipitated by an acute illness or trauma


and has a very high mortality rate?

A570.- Myxedema coma - (treat with IV levothyroxine)

Q571.- Labs reveal low free T4, increased TSH, and high titers for antibodies to
thyroperoxidase and thyroglobulin. Diagnosis?

A571.- Hashimoto s thyroiditis

Q572.- What type of thyroiditis will present with a very painful, tender, red th
yroid gland?

A572.- Suppurative thyroiditis - (rare disorder caused by pyogenic bacteria)

Q573.- Will a cancerous thyroid nodule be "cold" or "hot" on radionuclide scan?

A573.- Cold

Q574.- What hormone that is released from the zona glomerulosa of the adrenal co
rtex helps to regulate sodium balance?

A574.- Aldosterone

Q575.- What hormone controls the release of aldosterone?

A575.- Renin

Q576.- What hormone that is released from the zona fasciculata of the adrenal co
rtex helps to maintain physiologic integrity?

A576.- Cortisol

Q577.- What hormone controls the release of cortisol?

A577.- ACTH

Q578.- What disease is characterized by an excess of glucocorticoids?

A578.- Cushing s syndrome

Q579.- What is the most common cause of ACTH-dependent Cushing s syndrome?

A579.- ACTH-secreting pituitary tumor.

Q580.- What are the ACTH-dependent causes of Cushing s syndrome?

A580.- ACTH-secreting pituitary tumor, - small cell carcinoma of the lung (secre
tes ACTH), - endocrine tumors of foregut origin, - pheochromocytoma, - and ovari
an tumors

Q581.- What are the causes of ACTH-independent Cushing s syndrome?

A581.- Adrenal adenoma, - adrenal carcinoma, - and glucocorticoid administration

Q582.- What is the most common cause of chronic corticoadrenal insufficiency?

A582.- Autoimmune. - Other causes include tuberculosis and bilateral adrenal hem
orrhage

Q583.- Which glucocorticoid has the highest potency?- A. Prednisone- B. Hydrocor


tisone- C. Dexamethasone- D. Methylprednisone

A583.- C. Dexamethasone

Q584.- Testing reveals a low plasma renin activity and elevated 24-hour urine al
dosterone level. Diagnosis?

A584.- Hyperaldosteronism (Conn syndrome)

Q585.- What are the treatment options for Conn syndrome?

A585.- Adrenalectomy or lifelong spironolactone therapy

Q586.- What are the 2 primary hormones released from the posterior pituitary gla
nd?

A586.- Antidiuretic hormone (ADH) and Oxytocin

Q587.- What is the difference between gigantism and acromegaly?

A587.- Gigantism occurs during childhood, before closure of the epiphyses. - Acr
omegaly occurs after closure of the epiphyses.

Q588.- What is the most common cause of acromegaly?

A588.- Pituitary adenoma

Q589.- After surgery, what is the treatment for persistent acromegaly?

A589.- somatostatin analogs - (octreotide and lanreotide)

Q590.- Low blood pressure, weight loss, and abdominal pain associated with hypog
lycemia, hyponatremia, and hyperkalemia. Diagnosis?

A590.- Addison s disease (adrenal insufficiency)

Q591.- What disease is caused by deficiency of or resistance to vasopressin?

A591.- Diabetes insipidus

Q592.- What is the treatment of choice for central diabetes insipidus?

A592.- Desmopressin acetate, intranasally

Q593.- What is the initial treatment of choice for hyperprolactinemia?

A593.- Dopamine agonists - (cabergoline, bromocriptine, pergolide)

Q594.- What is the most common cause of hyperthyroidism?

A594.- Grave s Disease

Q595.- What is Grave s Disease?

A595.- Autoimmune disorder in which autoantibodies attach to TSH receptors & sti
mulate thyroid hyperfunctioning.

Q596.- A 30 yr old female presents with high fever, vomiting, diarrhea, tachycar
dia, restlessness and confusion. Labs: TSH - low, T3 and T4 are elevated. What i
s the condition? What is the initial treatment of choice?

A596.- Thyroid Storm- b-blockers (primarily propanolol)

Q597.- What is the drug of choice for pregnant/breast feeding women with hyperth
yroidism?

A597.- Propylthiouracil

Q598.- Which lab tests are used to confirm the dx of Hashimoto s Thyroiditis?

A598.- serum antithyroid peroxidase & antithyroglobulin antibodies

Q599.- What is the treatment for Hashimoto s thyroiditis?

A599.- lifetime replacement of thyroid hormone

Q600.- A 25 yr old female presents with painful glandular enlargement with pain
radiating to the ears, dysphagia, low-grade fever and malaise. Which type of thy
roiditis does she have & what is the treatment?

A600.- Subacute Thyroiditis- Tx - ASA

Q601.- Fever, pain, redness & a neck mass are present in _______ thyroiditis. Wh
at is the treatment?

A601.- Suppurative Thyroiditis- Tx - Abx, I/D

Q602.- What percentage of palpable thyroid nodules are malignant?

A602.- 0.05

Q603.- What is Chvostek s sign?

A603.- Facial muscle contraction after tapping the facial nerve

Q604.- What is Trousseau s phenomenon?

A604.- Carpal spasm with BP cuff inflation

Q605.- Which 2 drugs should be avoided in patients with hypoparathyroidism?

A605.- Phenothiazines- Furosemide

Q606.- What is the hallmark lab test of hyperparathyroidism?

A606.- Hypercalcemia >10.5 mg/dL

Q607.- What are the diagnostic lab values associated with DM I and II

A607.- Random plasma glucose >200 with classic sx; - or Fasting levels of 126 or
greater on more than one occasion

Q608.- A fasting glucose of 100-125 indicates ___________

A608.- Impaired fasting glucose

Q609.- A glucose of 140-199 two hrs after 75g oral glucose indicates __________-

A609.- Impaired glucose tolerance

Q610.- What is the MOA of Metformin?

A610.- Reduction of hepatic glucose production

Q611.- What are the acceptable glucose levels for DM pts?

A611.- 70-100 before meals & after overnight fast- <180 at 1 hr postprandial- <1
50 at 2 hrs postprandial

Q612.- What are the LDL and HDL goals for DM patients?

A612.- LDL <100- HDL >50

Q613.- What is Whipple s Triad?

A613.- Hx of hypoglycemic sx- Fasting blood glucose <40- Immediate recovery with
administration of glucose

Q614.- How is Addison s Disease diagnosed?

A614.- 1.) 8:00am plasma cortisol <3 ug/dL with elevated ACTH >200 pg/mL- 2.)Cos
yntropin stimulation test: serum cortisol increase <20 ug/dL after administratio
n of cosyntropin

Q615.- DHEA levels <1000 ng/mL ________ Addison s

A615.- excludes

Q616.- Buffalo hump, moon faces and supraclavicular pads are associated with ___
________

A616.- Cushing s Disease

Q617.- Excretion of free cortisol in the urine _______ in 24 hrs or greater than
_______ of cortisol per gram of creatinine is diagnostic of Cushing s

A617.- >125 mg/dL (free cortisol in urine)- 95 mcg (cortisol per gram of creatin
ine)

Q618.- In Cushing s disease overnight dexamethsone suppression will result in __


___________

A618.- plasma cortisol >10 ug/dL- (<5 ug/dL is normal)

Q619.- Which drugs can cause false postives in cortisol excretion & plasma corti
sol tests?

A619.- Rifampin - Phenytoin- Primidone- Phenobarbital- Carbamazapine- Fenofibrat


e- Estrogens- Pregnancy

Q620.- What is the treatment of choice for Cushing s Syndrome?

A620.- transsphenoidal resection of the pituitary adenoma & hydrocortisol replac


ement

Q621.- What is the treatment of choice for central DI and DI assoc with pregnanc
y?

A621.- Desmopressin acetate

Q622.- What is the treatment for nephrogenic DI?

A622.- Indomethacin +/- HCTZ, Desmopressin or Ameloride

Q623.- Milkman lines or Looser zones on x-ray indicate ________

A623.- Osteomalacia

Q624.- Patients of _______should be treated prophylactically for osteomalacia.

A624.- Phenytoin

Q625.- What is the treatment of choice for Paget s?

A625.- Prompt cyclic administration of bisphosphonates - alendronate, tiludronat


e, zoledronic acid, or pamidronate)

Q626.- Dietary cholesterol intake should not exceed ________ for patients with D
yslipidemia

A626.- 200 mg/daily

Q627.- What is the MOA of statins?

A627.- inhibition of rate-limiting enzyme in formation of cholestrol

Q628.- What is the MOA of fibric acids derivatives (gemfibrozil, fenofibrate)?

A628.- reduction of synthesis & increased breakdown of VLDL

Q629.- What is the MOA of Ezetimibe?

A629.- blocks intestinal absorption of dietary & biliary cholesterol

You might also like